ADULTS 3 EXAM 2

Pataasin ang iyong marka sa homework at exams ngayon gamit ang Quizwiz!

*The nurse is educating a patient and the family about different types of stabilization devices. Which statement by the patient indicates that the patient understands the benefit of using a halo fixation device instead of Gardner-Wells tongs?* 1. "I will have less pain if I use the halo device." 2. "The halo device will allow me to get out of bed." 3. "I am less likely to get an infection with the halo device." 4. "The halo device does not have to stay in place as long."

*2. "The halo device will allow me to get out of bed."* Rationale: A halo device will allow the patient to be mobile since it does not require weights like the Gardner-Wells tongs. The patient's pain level is not dependant on the type of stabilization device used. The patient does not have a great risk of infection with the Garnder-Wells tongs; both devices require pins to be inserted into the skull. The time required for stabilization is not dependant on the type of stabilization device used.

*A patient with a spinal cord injury at the T1 level complains of a severe headache and an "anxious feeling." Which is the most appropriate initial reaction by the nurse?* 1. Try to calm the patient and make the environment soothing. 2. Assess for a full bladder. 3. Notify the healthcare provider. 4. Prepare the patient for diagnostic radiography.

*2. Assess for a full bladder.* Rationale: Autonomic dysreflexia occurs in patients with injury at level T6 or higher, and is a life-threatening situation that will require immediate intervention or the patient will die. The most common cause is an overextended bladder or bowel. Symptoms include hypertension, headache, diaphoresis, bradycardia, visual changes, anxiety, and nausea. A calm, soothing environment is fine, though not what the patient needs in this case. The nurse should recognize this as an emergency and proceed accordingly. Once the assessment has been completed, the findings will need to be communicated to the healthcare provider.

*Which of the following is the priority nursing diagnosis for the patient who has undergone surgery for a spinal fusion?* 1. Acute Pain 2. Impaired Mobility 3. Risk for Infection 4. Risk for Injury

*2. Impaired Mobility* Rationale: The priority nursing diagnosis for a patient who has undergone a spinal fusion is Impaired Mobility, due to the assessment of the ABCs (airway, circulation, breathing). Impaired mobility can affect the patient's circulation, therefore affecting tissue perfusion and causing a risk for skin breakdown. Acute Pain is the next priority since it is an active diagnosis. Diagnoses with "risk for" do not take priority over active diagnoses.

*A hospitalized patient with a C7 cord injury begins to yell "I can't feel my legs anymore." Which is the most appropriate action by the nurse?* 1. Remind the patient of her injury and try to comfort her. 2. Call the healthcare provider and get an order for radiologic evaluation. 3. Prepare the patient for surgery, as her condition is worsening. 4. Explain to the patient that this could be a common, temporary problem.

*4. Explain to the patient that this could be a common, temporary problem.* Rationale: Spinal shock is a condition almost half the people with acute spinal injury experience. It is characterized by a temporary loss of reflex function below level of injury, and includes the following symptomatology: flaccid paralysis of skeletal muscles, loss of sensation below the injury, and possibly bowel and bladder dysfunction and loss of ability to perspire below the injury level. In this case, the nurse should explain to the patient what is happening.

*Which of the following nursing actions is appropriate for preventing skin breakdown in a patient who has recently undergone a laminectomy?* 1. Provide the patient with an air mattress. 2. Place pillows under patient to help patient turn. 3. Teach the patient to grasp the side rail to turn. 4. Use the log roll to turn the patient to the side.

*4. Use the log roll to turn the patient to the side.* Rationale: A patient who has undergone a laminectomy needs to be turned by log rolling to prevent pressure on the area of surgery. An air mattress will help prevent skin breakdown but the patient still needs to be turned frequently. Placing pillows under the patient can help take pressure off of one side but the patient still needs to change positions often. Teaching the patient to grasp the side rail will cause the spine to twist, which needs to be avoided.

*The patient with HHS presented with a glucose level of 800 mg/dL and is started on IV fluids and insulin. What action do you anticipate when the patient's glucose reaches 250 mg/dL?* A. Administer an intravenous (IV) solution with 5% dextrose. B. Administer sodium polystyrene sulfate (Kayexalate). C. Slow the IV infusion rate to 40 mL/hour. D. Assess cardiac monitoring for peaked T waves.

*A. Administer an intravenous (IV) solution with 5% dextrose.*

*The patient with HHS presented with a glucose level of 800 mg/dL and is started on IV fluids and insulin. What action do you anticipate when the patient's glucose reaches 250 mg/dL?* A. Administer an intravenous (IV) solution with 5% dextrose. B. Administer sodium polystyrene sulfate (Kayexalate). C. Slow the IV infusion rate to 40 mL/hour. D. Assess cardiac monitoring for peaked T waves.

*A. Administer an intravenous (IV) solution with 5% dextrose.* When blood glucose levels fall to approximately 250 mg/dL, IV fluids containing glucose are administered to prevent hypoglycemia. Kayexalate (for cation exchange) is used in the treatment of hyperkalemia, which causes peaked T waves on cardiac monitoring. In HHS hypokalemia may result from insulin moving the potassium intracellularly. Fluid replacement remains a priority, but it is given with dextrose. The infusion rate of 40 mL/hour keeps the vein open, but it is not the appropriate replacement rate. DICKS is for HYPERkalemia.

*The elderly patient with type 2 diabetes mellitus presents to the clinic with a fever and productive cough. The diagnosis of pneumonia is made. You notice tenting skin, deep tongue furrows, and vital signs of 110/80 mm Hg, 120 beats/minute, and 24 breaths/minute. What assessment is important for you to obtain?* A. Blood glucose B. Orthostatic blood pressures C. Urine ketones D. Temperature

*A. Blood glucose* HHS is typically seen with an INFECTION, such as pneumonia. The main presenting sign is a glucose level above 600 mg/dL. Enough evidence of dehydration already exists that orthostatic vital sign assessments are not a priority, and they are often inaccurate in the elderly due to poor vascular tone. Patients with HHS do not have elevated ketone levels, which is a key distinction between HHS and DKA. Temperature will eventually be taken but is often blunted in the elderly and diabetics. An infectious diagnosis has already been made. The glucose level for appropriate fluid and insulin treatment is the priority.

*You are caring for a patient hospitalized with an exacerbation of asthma and a herniated lumbar disk. Which breakfast choice should you encourage the patient to check on the breakfast menu?* A. Bran muffin B. Scrambled eggs C. Puffed rice cereal D. Buttered white toast

*A. Bran muffin* Each meal should contain one or more sources of fiber, which can reduce the risk of constipation and straining at stool, which increases back pain. Bran is typically a high-fiber food choice and is the appropriate selection from the menu

*The nurse in the cardiac care unit is caring for a patient receiving epinephrine. Which assessment criterion takes priority in the monitoring for adverse effects of this drug?* A. Cardiac rhythm B. Blood urea nitrogen C. Central nervous system (CNS) tremor D. Lung sounds

*A. Cardiac rhythm* Epinephrine can cause a number of adverse effects, including hypertensive crisis, dysrhythmias, angina, necrosis after extravasation, and hyperglycemia. Monitoring of the heart rhythm is essential to assess the patient for dysrhythmias.

*The patient with type 1 diabetes mellitus is diaphoretic and shaky, and he reports feeling very lightheaded. There is no readily accessible blood glucose monitoring equipment. What action should you take?* A. Give 8 oz of milk. B. Give 6 oz of juice with four packets of sugar in the juice. C. Wait to act until there is confirmation of the blood glucose level. D. Administer the patient's insulin.

*A. Give 8 oz of milk.*

Which of the following signs and symptoms in a patient with a T4 spinal cord injury should alert the nurse to the possibility of autonomic dysreflexia? A. Headache and rising blood pressure B. Irregular respirations and shortness of breath C. Decreased level of consciousness or hallucinations D. Abdominal distention and absence of bowel sounds

*A. Headache and rising blood pressure* Among the manifestations of autonomic dysreflexia are hypertension (up to 300 mm Hg systolic) and a throbbing headache. Respiratory manifestations, decreased level of consciousness, and gastrointestinal manifestations are not characteristic.

*The nurse realizes that beta2 receptor stimulation is differentiated from beta1 stimulation in that stimulation of beta2 receptors leads to which condition?* A. Increased bronchodilation B. Decreased uterine contractility C. Increased myocardial contractility D. Decreased blood flow to skeletal muscles

*A. Increased bronchodilation*

*The nurse is preparing to administer a dose of clonidine (Catapres). Which is the best description of the action of this drug?* A. It selectively activates alpha2 receptors in the central nervous system (CNS). B. It causes peripheral activation of alpha1 and alpha2 receptors. C. It depletes sympathetic neurons of norepinephrine. D. It directly blocks alpha and beta receptors in the periphery.

*A. It selectively activates alpha2 receptors in the central nervous system (CNS).*

A nurse is preparing to give bethanechol (Urecholine). What is an expected outcome of this drug? A. Nondistended bladder B. Increased heart rate and blood pressure C. Improved pulse oximetry reading D. Relief of cardiac rhythm problems

*A. Nondistended bladder*

A nurse is preparing to give bethanechol (Urecholine). What is an expected outcome of this drug? A. Nondistended bladder B. Increased heart rate and blood pressure C. Improved pulse oximetry reading D. Relief of cardiac rhythm problems

*A. Nondistended bladder* Bethanechol is a muscarinic agonist and therefore activates muscarinic receptors. This can lead to relaxation of the urinary sphincter muscles and increased voiding pressure. It also can cause bradycardia and hypotension, bronchoconstriction, and dysrhythmias in hyperthyroid patients.

*Which patient is at highest risk for hypokalemia?* A. Patient who has diabetic ketoacidosis who is receiving IV insulin B. Patient with chronic kidney disease on dialysis C. Patient with metabolic acidosis D. Patient with dehydration

*A. Patient who has diabetic ketoacidosis who is receiving IV insulin* Insulin causes extracellular potassium to move into the intercellular fluid. Treatment of diabetic ketoacidosis can cause hypokalemia because of the increased urination, shift of potassium with insulin therapy, and correction of acidosis. Ninety percent of daily potassium is excreted in the kidneys. Renal failure is one of the most common causes of hyperkalemia. Dehydration is more likely to lead to hyperkalemia; hypokalemia is a result of diuresis.

*Which nursing intervention is most appropriate when turning a patient after spinal surgery?* A. Placing a pillow between the patient's legs and turning the body as a unit B. Having the patient turn to the side by grasping the side rails to help turn over C. Elevating the head of bed 30 degrees and having the patient extend the legs while turning D. Turning the patient's head and shoulders and then the hips, keeping the patient's body centered in the bed Rationale

*A. Placing a pillow between the patient's legs and turning the body as a unit* Placing a pillow between the legs and turning the patient as a unit (log rolling) helps to keep the spine in good alignment and to reduce pain and discomfort after spinal surgery.

*The nurse is caring for a patient receiving atropine (Sal-Tropine). Which is a therapeutic indication for giving this drug?* A. Use as a preanesthesia medication B. Treatment of tachycardias C. Prevention of urinary retention D. Reduction of intraocular pressure in glaucoma

*A. Use as a pre anesthesia medication* Atropine is an anticholinergic drug. Why would they give it before surgery? It decreases secretions and prevents aspiration. An anticholinergic drug would cause tachycardia not treat it, would cause urinary prevention not treat it and would increase intraocular pressure, not relief it.

*The nurse is caring for a patient receiving atropine (Sal-Tropine). Which is a therapeutic indication for giving this drug?* A. Use as a preanesthesia medication B. Treatment of tachycardias C. Prevention of urinary retention D. Reduction of intraocular pressure in glaucoma

*A. Use as a pre anesthesia medication* Atropine is a muscarinic antagonist and can help *prevent dangerous bradycardia during surgery*. It often is administered before the induction of anesthesia. Its side effects may include urinary retention, constipation, and tachycardia.

*The patient received regular insulin 10 units subcutaneously at 8:30 pm for a blood glucose level of 253 mg/dl. The nurse plans to monitor this patient for signs of hypoglycemia at which of the following peak action times?* A. 9:00 pm to 10:30 pm B. 10:30 pm to 11:30 pm C. 12:30 am to 1:30 am D. 2:30 am to 4:30 am

*B. 10:30 pm to 11:30 pm* Regular insulin exerts peak action in 2 to 3 hours, making the patient most at risk for hypoglycemia between 10:30 pm and 11:30 pm.

*The nurse is caring for a group of patients who are all receiving anticholinergic drugs. In which patient is an anticholinergic drug contraindicated?* A. A 60-year-old woman with an overactive bladder (OAB) B. A 72-year-old man with glaucoma C. A 45-year-old woman with peptic ulcer disease (PUD) D. A 26-year-old man being prepared for surgery today

*B. A 72-year-old man with glaucoma* Anticholinergic drugs tend to cause adverse effects of dry mouth, blurred vision, elevation of intraocular pressure, urinary retention, constipation, anhidrosis, tachycardia, and asthma. They are contraindicated in patients with glaucoma because of the potential danger of increased intraocular pressure. Anticholinergics are used to treat OAB and are used as preanesthetic agents. They are not contraindicated in PUD.

*The nurse is caring for a group of patients who are all receiving anticholinergic drugs. In which patient is an anticholinergic drug contraindicated?* A. A 60-year-old woman with an overactive bladder (OAB) B. A 72-year-old man with glaucoma C. A 45-year-old woman with peptic ulcer disease (PUD) D. A 26-year-old man being prepared for surgery today

*B. A 72-year-old man with glaucoma* Anticholinergics increase intra-ocular pressure so they would want a cholinergic drug which decreases intra-ocular pressure

*The patient is managed with NPH and regular insulin injections before breakfast and before dinner. When is the patient most likely to have a hypoglycemic reaction?* A. After breakfast B. Before lunch C. During lunch D. After lunch

*B. Before lunch* The regular insulin peak occurs about 2 to 3 hours with a duration of 5 to 6 hours. If too much insulin or not enough food is given, the most likely time of hypoglycemia is before lunch, when the regular insulin is still present, the NPH has its onset, and the breakfast food has been metabolized.

*The nurse is caring for a patient newly diagnosed with HIV. The patient asks what would determine the actual development of AIDS. The nurse's response is based on the knowledge that which of the following is a diagnostic criterion for AIDS?* A. Presence of HIV antibodies B. CD4+ T cell count <200/µl C. White blood cell count <5000/µl D. Presence of oral hairy leukoplakia

*B. CD4+ T cell count <200/µl* Diagnostic criteria for AIDS include a CD4+ T-cell count <200/µl and/or the development of specified opportunistic infections, cancers, wasting syndrome, or dementia. The other options may be found in patients with HIV disease, but do not define the advancement of the disease to AIDS.

*The patient with type 1 diabetes arrives in the emergency department with a glucose level of 390 mg/dL and positive result for ketones. Vital signs are 110/70 mm Hg, 120 beats/minute, and 28 deep, sighing respirations/minute. What is the priority need for the patient?* A. Oxygen B. Intravenous (IV) fluids C. Albuterol (Ventolin) D. Metformin (Glucophage)

*B. Intravenous (IV) fluids*

*The patient with type 1 diabetes arrives in the emergency department with a glucose level of 390 mg/dL and positive result for ketones. Vital signs are 110/70 mm Hg, 120 beats/minute, and 28 deep, sighing respirations/minute. What is the priority need for the patient?* A. Oxygen B. Intravenous (IV) fluids C. Albuterol (Ventolin) D. Metformin (Glucophage)

*B. Intravenous (IV) fluids* A patient in diabetic ketoacidosis (DKA) needs IV fluids and insulin to stop the tissue breakdown resulting in ketone bodies and acidosis. The initial goal is fluid and electrolyte balance. Kussmaul respirations indicate the body is attempting to compensate by blowing off the carbon dioxide, but it is ineffective as long as the body continues to break down the ketone bodies and remains in metabolic acidosis. The issue is not respiratory insufficiency, and a bronchodilator is not indicated. Diabetic ketoacidosis occurs in type 1 diabetes and requires insulin; the pancreas no longer has the ability to respond to oral hypoglycemic medication.

*The nurse is preparing to give physostigmine. What best describes the action of this drug?* A. It inhibits acetylcholine at all cholinergic synapses. B. It prevents inactivation of acetylcholine. C. It prevents activation of muscarinic receptors. D. It stimulates activation of adrenergic receptors

*B. It prevents inactivation of acetylcholine.* This treats anti-cholinergic crisis. Neostigmine is a cholinesterase inhibitor. As such, it prevents the inactivation of acetylcholine, allowing it to linger at the synapses. It lacks selectivity and thus intensifies transmission at all cholinergic junctions. This drug is used for patients in anticholinergic crisis due to overdose or toxicity of an anticholinergic drug. Patients on first generation antipsychotics experience suppression of DA and an increase in Acetylcholine. This causes the movement disorders. This is treated with an anticholinergic that decreases acetylcholine in the muscles but also the brain, causing confusion. If too much is given they can go into anticholinergic crisis. Prostigmin is given to help with this.

The nurse is beginning to teach a diabetic patient about vascular complications of diabetes. Which of the following information would be appropriate for the nurse to include? A. Macroangiopathy does not occur in type 1 diabetes but rather in type 2 diabetics who have severe disease. B. Microangiopathy is specific to diabetes and most commonly affects the capillary membranes of the eyes, kidneys, and skin. C. Renal damage resulting from changes in large- and medium-sized blood vessels can be prevented by careful glucose control. D. Macroangiopathy causes slowed gastric emptying and the sexual impotency experienced by a majority of patients with diabetes.

*B. Microangiopathy is specific to diabetes and most commonly affects the capillary membranes of the eyes, kidneys, and skin.* Microangiopathy occurs in diabetes mellitus. When it affects the eyes, it is called diabetic retinopathy. When kidneys are affected, the patient has nephropathy. When the skin is affected, it can lead to diabetic foot ulcers. Sexual impotency and slowed gastric emptying result from microangiopathy. Macroangiopathy can occur in either type 1 or type 2 diabetes.

Which label most aptly describes the drug atropine (Sal-Tropine)? A. Cholinergic B. Parasympatholytic C. Muscarinic agonist D. Parasympathomimetic

*B. Parasympatholytic* Atropine is a muscarinic antagonist agent. Other terms for this agent are parasympatholytic, antimuscarinic, muscarinic blocker, and anticholinergic.

*Why does the nurse anticipate administering metoprolol (Lopressor) rather than propranolol (Inderal) for diabetic patients who need a beta-blocking agent?* A. Metoprolol is less likely to cause diabetic nephropathy. B. Propranolol causes both beta1 and beta2 blockade. C. Metoprolol helps prevent retinopathy in individuals with diabetes. D. Propranolol is associated with a higher incidence of foot ulcers.

*B. Propranolol causes both beta1 and beta2 blockade.* Metoprolol is a second-generation beta blocker and as such is more selective. At therapeutic doses, it causes less bronchoconstriction and suppression of glycogenolysis, which can cause problems in diabetic patients. Propranolol blocks both beta1 and beta2 receptors. *Blocking BETA 2 inhibits glycogenolysis in the liver.*

A nurse is reviewing laboratory reports of a client who has hyperglycemic-hyperosmolar state (HHS). Which of the following is an expected finding? A. Serum pH 7.2 B. Serum osmolarity 350 mOsm/L C. Serum potassium 3.8 mg/dL D. Serum creatinine 0.8 mg/dL

*B. Serum osmolarity 350 mOsm/L*

*What is a typical finding of hyperosmolar hyperglycemic syndrome (HHS)?* A. Occurs in type 1 diabetes as the presenting symptom B. Slow onset resulting in a blood glucose level greater than 600 mg/dL C. Ketone bodies higher than 4+ in urine D. Signs and symptoms of diabetes insipidus

*B. Slow onset resulting in a blood glucose level greater than 600 mg/dL* HHS has a slower onset than diabetic ketoacidosis. HHS is often related to impaired thirst sensation, inadequate fluid intake, or functional inability to replace fluids. Because of the slower onset, the blood glucose levels can be quite high (more than 600 mg/dL) before diagnosis. HHS is seen in type 2 diabetics, and there is enough circulating insulin to prevent ketoacidosis. Diabetes insipidus is related to inadequate antidiuretic hormone secretion or kidney response with dilute, frequent urination. It is not related to HHS.

*The nurse is caring for a patient admitted with a spinal cord injury following a motor vehicle accident. The patient exhibits a complete loss of motor, sensory, and reflex activity below the injury level. The nurse recognizes this condition as which of the following?* A. Central cord syndrome B. Spinal shock syndrome C. Anterior cord syndrome D. Brown-Séquard syndrome

*B. Spinal shock syndrome* About 50% of people with acute spinal cord injury experience a temporary loss of reflexes, sensation, and motor activity that is known as spinal shock. Central cord syndrome is manifested by motor and sensory loss greater in the upper extremities than the lower extremities. Anterior cord syndrome results in motor and sensory loss but not reflexes. Brown-Séquard syndrome is characterized by ipsilateral loss of motor function and contralateral loss of sensory function.

*What is important to teach a patient who has intervertebral disk disease?* A. Sleep in a prone position at night. B. Stop smoking. C. Take an analgesic before work involving twisting. D. Exhale when lifting.

*B. Stop smoking.* Nicotine decreases circulation to the vertebral disks, and there is a causal relationship between smoking and some types of low back pain. Sleeping should be done in a side-lying position or on the back. The prone position places excessive stress on the lower back. Activities that put strain on the spine should be avoided, including twisting. Exhaling when lifting keeps intracranial pressure within normal levels and is not a priority intervention for disk disease.

*One month after a spinal cord injury, which finding is most important for you to monitor?* A. Bladder scan indicates 100 mL. B. The left calf is 5 cm larger than the right calf. C. The heel has a reddened, nonblanchable area. D. Reflux bowel emptying.

*B. The left calf is 5 cm larger than the right calf.*

*The nurse is caring for a patient receiving propranolol (Inderal). Which finding is most indicative of an adverse effect of this drug?* A. A heart rate of 100 beats per minute B. Wheezing C. A glucose level of 180 mg/dL D. Urinary urgency

*B. Wheezing* Beta blockers, such as propranolol, are known to cause bronchoconstriction (block beta2), which could manifest as wheezing. Other adverse effects could include bradycardia, atrioventricular (AV) heart block, heart failure, rebound cardiac excitation, inhibition of glycogenolysis, and potential central nervous system (CNS) effects.

*The nurse is caring for a 60-year-old woman who has been prescribed oxybutynin (Ditropan) for the treatment of overactive bladder (OAB). Which statement by the nurse will be the most helpful to include in the teaching plan?* A. "You may experience a slower heart rate. Call your doctor if it is below 60." B. "Ditropan is very effective. Most patients experience significant relief." C. "Sip on water and hard candy to help with the problem of dry mouth." D. "Antihistamines, such as Benadryl, can help with some of the side effects of Ditropan."

*C. "Sip on water and hard candy to help with the problem of dry mouth."* Oxybutynin is an anticholinergic drug that commonly causes dry mouth. Other side effects include constipation, tachycardia, urinary hesitancy, urinary retention, and visual disturbances. Oxybutynin is only moderately effective (30% better than placebo). It should not be taken with other drugs with anticholinergic properties, such as antihistamines, because of the additive anticholinergic effects.

*A nurse is caring for a client who has a new diagnosis of HIV infection and is beginning combination oral NRTIs (abacavir, lamivudine, and zidovudine [Trizivir]). The client asks how medications work to treat HIV. Which of the following responses by the nurse is appropriate?* A. "These medications work by blocking HIV entry into cells." B. "These medications work by weakening the cell wall of the HIV virus." C. "These medications work by inhibiting enzymes to prevent HIV replication." D. "These medications work by preventing protein synthesis within the HIV cell.

*C. "These medications work by inhibiting enzymes to prevent HIV replication."*

*The nurse is caring for several patients. For which patient is propranolol (Inderal) most likely to be contraindicated?* A. A 30-year-old woman with cardiac dysrhythmias B. A 48-year-old man with hypertension C. A 60-year-old woman with diabetes D. A 72-year-old man with angina

*C. A 60-year-old woman with diabetes* *Propranolol inhibits glycogenolysis* and thus can produce hypoglycemia, which can cause problems in patients with diabetes. It also *suppresses tachycardia, which is an important warning sign of hypoglycemia* in diabetic patients. It is safe to use propranolol in dysrhythmias, hypertension, and angina.

*The nurse is providing care for a patient who has been living with HIV for several years. Which of the following assessment findings most clearly indicates an acute exacerbation of the disease?* A. A new onset of polycythemia B. Presence of mononucleosis-like symptoms C. A sharp decrease in the patient's CD4+ count D. A sudden increase in the patient's WBC count

*C. A sharp decrease in the patient's CD4+ count* A decrease in CD4+ count signals an exacerbation of the severity of HIV. Polycythemia is not characteristic of the course of HIV. A patient's WBC count is very unlikely to suddenly increase, with decreases being typical. Mononucleosis-like symptoms such as malaise, headache, and fatigue are typical of early HIV infection and seroconversion.

*Which of the following nursing diagnoses is likely to be a priority in the care of a patient with myasthenia gravis (MG)?* A. Acute confusion B. Bowel incontinence C. Activity intolerance D. Disturbed sleep pattern

*C. Activity intolerance* The primary feature of MG is fluctuating weakness of skeletal muscle due to low acetylcholine. Bowel incontinence and confusion are unlikely signs of MG, and although sleep disturbance is likely, activity intolerance is usually of primary concern.

*The patient has type 1 diabetes mellitus and is found unresponsive with cool and clammy skin. What action is a priority?* A. Obtain a serum glucose level. B. Give hard candy under the tongue. C. Administer glucagon per standing order. D. Notify the health care provider.

*C. Administer glucagon per standing order.* The patient has signs and symptoms of hypoglycemia for which treatment should be the priority. Glucagon stimulates a strong hepatic response to convert glycogen to glucose and therefore makes glucose rapidly available. Waiting for a serum result (up to an hour) is improper because brain cells continue to die from a lack of glucose. Nothing solid should be placed in the mouth when the patient has an altered level of consciousness and can aspirate. With obvious symptoms, emergent treatment takes priority over notifying the health care provider.

*Transmission of HIV from an infected individual to another most commonly occurs as a result of?* A. Perinatal transmission. B. Sharing drug-using equipment. C. Anal or vaginal sexual intercourse. D. Low levels of virus in the blood.

*C. Anal or vaginal sexual intercourse.*

*A client diagnosed with Alzheimer's disease is prescribed donepezil (Aricept) for treatment. The client has a medical history of osteoarthritis and GERD. For what possible complications of this drug therapy should the nurse assess in the client?* A. Tachycardia B. Bradycardia C. Aspiration D. Muscle pain

*C. Aspiration*

*The nurse is caring for a patient with myasthenia gravis who is beginning a new prescription of pyridostigmine (Mestinon), 75 mg PO twice daily. What is the most important initial nursing action?* A. Obtain a measurement of the plasma level of neostigmine B. Teach the patient to wear a Medic Alert bracelet C. Assess the patient's ability to swallow D. Check the patient's deep tendon reflexes (DTRs)

*C. Assess the patient's ability to swallow* Patients with myasthenia gravis have decreased muscle strength and may have impaired swallowing. If swallowing is insufficient, the medication must be given by the parenteral route. Plasma levels are not as important when beginning medication. Neostigmine typically is dosed to symptoms. The patient should be taught to wear a Medic Alert bracelet and may need to have DTRs monitored, but assessing swallowing is a higher priority.

*Which condition is an indication for the use of succinylcholine (Anectine)?* A. Pain relief after major surgery B. Loss of consciousness during surgery C. Facilitation of endotracheal intubation D. Relief of status epilepticus

*C. Facilitation of endotracheal intubation* inhibits gag reflex due to neuromuscular paralysis. Its an anti-cholinergic so it blocks ACh at nicotinic receptors. Muscles use Ach. Same reason they MG with cholinergic drugs. Muscles are weka so must be activated.

*The patient had a hypoglycemic episode and is treated with a concentrated glucose oral tablet. Fifteen minutes later the capillary glucose level (Accu-Check) is 150 mg/dL. What action should you take?* A. Administer a second bolus of glucose solution. B. Administer regular insulin per sliding scale. C. Have the patient eat peanut butter and toast. D. Obtain a serum glucose level.

*C. Have the patient eat peanut butter and toast.*

*What is the correct teaching regarding oral antidiabetic medications?* A. Double the glipizide (Glucotrol) dose if consuming alcohol. B. Hold all antidiabetic medication if vomiting is related to the flu. C. Hold metformin 48 hours before a procedure with contrast medium. D. Acarbose (Precose) is taken 2 hours after meals.

*C. Hold metformin 48 hours before a procedure with contrast medium.*

*The nurse realizes that beta1 receptor stimulation is differentiated from beta2 stimulation in that stimulation of beta1 receptors leads to which condition?* A. Increased bronchodilation B. Decreased uterine contractility C. Increased myocardial contractility D. Decreased blood flow to skeletal muscles

*C. Increased myocardial contractility*

*The nurse is reviewing drugs on the emergency cart with regard to their therapeutic action. Which medications can help initiate heart contraction during a cardiac arrest?* A. Topical phenylephrine B. Subcutaneous terbutaline C. Intravenous epinephrine D. Inhaled albuterol

*C. Intravenous epinephrine* When beta1 receptors are activated, cardiac contraction is stimulated. When a beta1 agonist is indicated, epinephrine (given IV) is the preferred drug. Beta1 agonist drugs (e.g., epinephrine), which are kept on the emergency cart, may be injected directly into the heart during an arrest. Alpha1 activation causes hemostasis, nasal decongestion, vasoconstriction, and mydriasis. Alpha2 activation reduces sympathetic outflow to the heart and blood vessels and relieves severe pain. Beta2 activation causes bronchodilation and relaxation of uterine smooth muscle. Topical phenylephrine is helpful for nasal congestion. Terbutaline is indicated for preterm labor and/or asthma. Albuterol is indicated for asthma.

*What is a finding in DKA that is not seen in hyperosmolar hyperglycemic syndrome (HHS)?* A. Glucose level above 400 mg/dL B. Hyperkalemia C. Ketones in blood D. Urine output of 30 mL/hr

*C. Ketones in blood* The main difference between the two conditions is that ketone bodies are ABSENT or minimal in HHS because the body has enough insulin to prevent ketoacidosis. Both have high glucose levels, although the level in HHS tends to be higher (above 600 mg/dL). Hypokalemia is possible in both, although it is more likely and serious in DKA. Urine output of 30 mL/hr is normal obligatory output; both conditions are likely to have dehydration and decreased output.

*A nurse is preparing to administer IV fluids to a client who has diabetic ketoacidosis. Which of the following is an appropriate nursing action?* A. Administer an IV infusion of regular insulin at 0.3 unit/kg/hr. B. Administer an IV infusion of 0.45% sodium chloride. C. Rapidly administer an IV infusion of 0.9% sodium chloride. D. Add glucose to the IV infusion when serum glucose is 350 mg/dL

*C. Rapidly administer an IV infusion of 0.9% sodium chloride.* 1000 mL/hr is rapid. This is followed by giving 0.45% NaCl. Then potassium is replaced before giving insulin.

*A patient is taking a Sulfonylurea and a beta-adrenergic medication. A nurse is teaching hypoglycemia awareness and should warn the patient about the absence of which symptom?* A. Vomiting B. Muscle cramps C. Tachycardia D. Chills

*C. Tachycardia* Sulfonylureas can cause hypoglycemia and beta-blockers mask the early symptoms of hypoglycemia, which tachycardia is one. Glipizide is a sulfonylurea oral hypoglycemic medication that acts to promote insulin release from the pancreas. Beta-adrenergic blockers can mask early signs of sympathetic system responses to hypoglycemia; the most important of these is tachycardia, which is the most common adverse effect of glipizide. Vomiting, muscle cramps, and chills are not symptoms of activation of the sympathetic nervous system that arise when glucose levels fall. SNS activated when glucose falls because its trying to STIMULATE Alpha 2 receptors, part of the adrenergic system that blocks release of insulin from pancreas to increase blood glucose.

*A patient with a C7 spinal cord injury undergoing rehabilitation tells you he must have the flu because he has a bad headache and nausea. What is your initial action?* A. Call the physician. B. Check the patient's temperature. C. Take the patient's blood pressure. D. Elevate the head of the bed to 90 degrees.

*C. Take the patient's blood pressure.*

*What happens after infection with HIV?* A. The virus replicates mainly in B lymphocytes before spreading to CD4+ T cells. B. Infection of monocytes may occur, but antibodies quickly destroy these cells. C. The immune system is impaired predominantly by the eventual widespread destruction of CD4+ T cells. D. A long period of dormancy develops, during which HIV cannot be found in the blood, and there is little viral replication.

*C. The immune system is impaired predominantly by the eventual widespread destruction of CD4+ T cells.*

*The patient is admitted with intractable pain caused by a herniated disk at L5-S1. What assessment requires you to contact the health care provider?* A. Positive straight-leg-raising test B. Pain that radiates down the leg C. Urinary incontinence D. Ankle reflex of 1+

*C. Urinary incontinence* Multiple nerve root (cauda equina) compression due to a herniated disk manifests as bowel or bladder incontinence or impotence, and it is a medical emergency. Other options are expected symptoms or signs seen of a herniated dis

A diabetic patient has a serum glucose level of 824 mg/dL (45.7 mmol/L) and is unresponsive. After assessment of the patient, you suspect DKA rather than HHS based on the finding of A. polyuria. B. severe dehydration. C. rapid, deep respirations. D. decreased serum potassium.

*C. rapid, deep respirations.* Signs and symptoms of DKA include manifestations of dehydration, such as poor skin turgor, dry mucous membranes, tachycardia, and orthostatic hypotension. Early symptoms may include lethargy and weakness. As the patient becomes severely dehydrated, the skin becomes dry and loose, and the eyeballs become soft and sunken. Abdominal pain is another symptom of DKA that may be accompanied by anorexia and vomiting. Kussmaul respirations (rapid, deep breathing associated with dyspnea) are the body's attempt to reverse metabolic acidosis through exhalation of excess carbon dioxide. Acetone is detected on the breath as a sweet, fruity odor. Laboratory findings include a blood glucose level of more than 250 mg/dL, arterial blood pH greater than 7.30, serum bicarbonate level more than 15 mEq/L, and moderate to large levels of ketones in the urine or blood.

The nurse is caring for several patients. For which patient is propranolol (Inderal) most likely to be contraindicated? A. A 30-year-old woman with cardiac dysrhythmias B. A 48-year-old man with hypertension C. A 72-year-old man with angina D. A 60-year-old woman with diabetes

*D. A 60-year-old woman with diabetes* Propranolol *inhibits glycogenolysis* and thus can produce hypoglycemia, which can cause problems in patients with diabetes. It also *suppresses tachycardia*, which is an important warning sign of hypoglycemia in diabetic patients. It is safe to use propranolol in dysrhythmias, hypertension, and angina.

A nurse prepares to administer a new prescription for bethanechol (Urecholine). Which information in the patient's history should prompt the nurse to consult with the prescriber before giving the drug? A. Constipation B. Hypertension C. Psoriasis D. Asthma

*D. Asthma* Muscarinic agonists induce bronchospasm, which would cause problems for a patient with a history of asthma.

*Which of the following is the mechanism of action for Metformin?* A. Decreases glucose production by the liver B. Increases glucose uptake by muscle and adipose tissue C. Decreases absorption of glucose from GI tract D. All of the above

*D. BUT the main effect is to decrease glucose production by the liver (MET-FORM-IN decreases the glucose that the liver's FORM-IN)*

*Which symptom is the most indicative of muscarinic poisoning?* A. Constipation B. Heart rate of 140 beats per minute C. Blood pressure of 180/110 mm Hg D. Blurred vision

*D. Blurred vision* Muscarinic poisoning can result from overdose of muscarinic agonists or cholinesterase inhibitors or from ingestion of certain mushrooms. The symptoms include profuse salivation, lacrimation, visual disturbances, bronchospasm, diarrhea, bradycardia, and hypotension.

*A client is given epinephrine (Adrenalin), an adrenergic agonist (sympathomimetic). The nurse should monitor the client for which condition?* A. Decreased pulse B. Pupil constriction C. Bronchial constriction D. Increased blood pressure

*D. Increased blood pressure*

*When planning care for a patient with a C5 spinal cord injury, which nursing diagnosis is the highest priority?* A. Risk for impairment of tissue integrity caused by paralysis B. Altered patterns of urinary elimination caused by quadriplegia C. Altered family and individual coping caused by the extent of trauma D. Ineffective airway clearance caused by high cervical spinal cord injury

*D. Ineffective airway clearance caused by high cervical spinal cord injury* Maintaining a patent airway is the most important goal for a patient with a high cervical fracture. Although all of these are appropriate nursing diagnoses for a patient with a spinal cord injury, respiratory needs are always the highest priority. Remember the ABCs.

*What best describes the rationale for using pyridostigmine (Mestinon) in the treatment of myasthenia gravis?* A. It promotes neuromuscular blockade in the periphery. B. It promotes emptying of the bladder and sphincter relaxation. C. It reduces intraocular pressure and protects the optic nerve. D. It increases the force of skeletal muscle contraction.

*D. It increases the force of skeletal muscle contraction.* Neostigmine is a cholinesterase inhibitor; therefore, at therapeutic doses it increases the force of contraction of skeletal muscles. Myasthenia gravis is a neuromuscular disease characterized by muscle weakness.

Which statement correctly describes a patient with type 2 diabetes who is admitted to the hospital with pneumonia? A. Must receive insulin therapy to prevent the development of ketoacidosis. B. Has islet cell antibodies that have destroyed the ability of the pancreas to produce insulin C. Has minimal or absent endogenous insulin secretion and requires daily insulin injections D. May have sufficient endogenous insulin to prevent ketosis but is at risk for hyperosmolar hyperglycemic syndrome (HHS).

*D. May have sufficient endogenous insulin to prevent ketosis but is at risk for hyperosmolar hyperglycemic syndrome (HHS).* WITH PNEUMONIA and TYPE 2 DIABETES THINK HHS! HHS is a life-threatening syndrome that can occur in the patient with diabetes who is able to produce enough insulin to prevent diabetic ketoacidosis (DKA) but not enough to prevent severe hyperglycemia, osmotic diuresis, and extracellular fluid depletion.

*The patient in the emergency department is diagnosed with diabetic ketoacidosis. Which laboratory value is essential for you to monitor?* A. Magnesium (Mg) B. Hemoglobin (Hb) C. White blood cells (WBCs) D. Potassium (K)

*D. Potassium (K)*

*The patient in the emergency department is diagnosed with diabetic ketoacidosis. Which laboratory value is essential for you to monitor?* A. Magnesium (Mg) B. Hemoglobin (Hb) C. White blood cells (WBCs) D. Potassium (K)

*D. Potassium (K)* Even if the patient has normal potassium levels, there can be significant hypokalemia when insulin is administered as it pushes the serum potassium intracellularly. This can lead to life-threatening hypokalemia. The other options are not as significant. H+ pushes K+ out INsulin pushes K+ IN

*The patient arrives in the emergency department from a motor vehicle accident, during which the car ran into a tree. The patient was not wearing a seat belt, and the windshield is shattered. What action is most important for you to do?* A. Determine if the patient lost consciousness. B. Assess the Glasgow Coma Scale (GCS) score. C. Obtain a set of vital signs. D. Use a logroll technique when moving the patient.

*D. Use a logroll technique when moving the patient.* When the head hits the windshield with enough force to shatter it, you must assume neck or cervical spine trauma occurred and you need to maintain spinal precautions. This includes moving the patient in alignment as a unit or using a logroll technique during transfers. The other options are important and are done after spinal precautions are applied

*The patient presents to the emergency department with a glucose level of 400 mg/dL, ketone result of 2+, and rapid respirations with a fruity odor. What finding do you anticipate?* A. Urine specific gravity below 1.005 B. High sodium bicarbonate levels C. Low blood urea nitrogen (BUN) level D. pH below 7.30 STATE WHY IT ISN'T THE OTHER OPTIONS

*D. pH below 7.30* NOT A-dehydration would cause a urine specific gravity closer to 1.030-concentrated. NOT B-bicarbonate would be low (below 24) NOT C-BUN would be elevated from dehydration (above 25)

*The patient presents to the emergency department with a glucose level of 400 mg/dL, ketone result of 2+, and rapid respirations with a fruity odor. What finding do you anticipate?* A. Urine specific gravity below 1.005 B. High sodium bicarbonate levels C. Low blood urea nitrogen (BUN) level D. pH below 7.30

*D. pH below 7.30* The patient is in metabolic acidosis, which is a pH below 7.35. Dehydration results in a high urine specific gravity (at the upper end of the normal range, or above 1.025 to 1.030). Sodium bicarbonate levels are low in metabolic acidosis. The dehydration that occurs with DKA elevates the BUN level.

*A patient is found non-responsive and has a blood glucose of 50 mg/dL. There is IV access readily available. What do you do?*

*Give IV 50% Dextrose*

*If a patient is severely hypoglycemic and is responsive but NPO what do you give if there is no IV access?*

*IM Glucagon (not used a lot)*

MY METFORMIN RYHME MNEMONIC:

*MET-FOR-MIN decreases glucose the livers FORM-IN (major effect), decreases the glucose the guts ABSORB-IN, and increases how much muscle and fat are STORE-IN (minor effect)*

Two days following a self-managed hypoglycemic episode at home, the patient tells the nurse that his blood glucose levels since the episode have been between 80 and 90 mg/dL. The best response by the nurse is? a. "That is a good range for your glucose levels." b. "You should call your health care provider because you need to have your insulin increased." c. "That level is too low in view of your recent hypoglycemia, and you should increase your food intake." d. "You should only take half your insulin dosage for the next few days to get your glucose level back to normal

*a. "That is a good range for your glucose levels."*

*A patient with diabetes calls the clinic because she is experiencing nausea and flulike symptoms. The nurse advises the patient to?* a. administer the usual insulin dosage. b. hold fluid intake until the nausea subsides. c. come to the clinic immediately for evaluation and treatment. d. monitor the blood glucose every 1 to 2 hours and call if the glucose rises over 150 mg/dL (8.3 mmol/L

*a. administer the usual insulin dosage.*

*A patient with diabetes calls the clinic because she is experiencing nausea and flulike symptoms. The nurse advises the patient to?* a. administer the usual insulin dosage. b. hold fluid intake until the nausea subsides. c. come to the clinic immediately for evaluation and treatment. d. monitor the blood glucose every 1 to 2 hours and call if the glucose rises over 150 mg/dL (8.3 mmol/L).

*a. administer the usual insulin dosage.* *REMEMBER THAT DURING STRESS -OR- ILLNESS counter-regualrtory hormones will raise blood glucose so its important to take insulin to avoid DKA ALSO: BGL check every 4 hours and call provider if >240 mg/dL

*A week following a spinal cord injury at T2, a patient experiences movement in his leg and tells the nurse he is recovering some function. The nurse's best response to the patient is?* a. "It is really still too soon to know if you will have a return of function." b. "That could be a really positive finding. Can you show me the movement?" c. "That's wonderful. We will start exercising your legs more frequently now." d. "I'm sorry, but the movement is only a reflex and does not indicate normal function."

*b. "That could be a really positive finding. Can you show me the movement?"*

*A patient is admitted to the emergency department with a spinal cord injury at the level of T2. Which of the following findings is of most concern to the nurse?* a. SpO2 of 92% b. HR of 42 beats/min c. Blood pressure of 88/60 d. Loss of motor and sensory function in arms and legs

*b. HR of 42 beats/min* T1-L4 affects the sympathetic nervous system. Above T6 is loss of sympathetic nervous system function and a common landmark for Autonomic Dysreflexia when there is a stimulus below the level of injury. When a stimulus below the level of injury occurs hypertension stimulates baroreceptors that causes bradycardia.

*A laminectomy and spinal fusion are performed on a patient with a herniated lumbar intervertebral disk. During the postoperative period, which of the following patient findings is of most concern to the nurse?* a. Paralytic ileus b. Urinary incontinence c. Greater pain at the graft site than at the lumbar incision site d. Leg and arm movement and sensation unchanged from preoperative stat

*b. Urinary incontinence* NERVE DAMAGE.

*A patient with type 1 diabetes uses 20 U of 70/30 neutral protamine Hagedorn (NPH/regular) in the morning and at 6:00 pm. When teaching the patient about this regimen, the nurse stresses that?* a. hypoglycemia is most likely to occur before the noon meal. b. a set meal pattern with a bedtime snack is necessary to prevent hypoglycemia. c. flexibility in food intake is possible because insulin is available 24 hours/day. d. pre-meal glucose checks are required to determine needed changes in daily dosing.

*b. a set meal pattern with a bedtime snack is necessary to prevent hypoglycemia.*

*Treatment with two nucleoside reverse transcriptase inhibitors (NRTIs) and a protease inhibitor (PI) is prescribed for a patient with HIV infection. The patient asks why so many drugs are necessary for treatment. The nurse explains that the primary rationale for combination therapy is that?* a. cross-resistance between specific antiretroviral drugs is reduced when drugs are given in combination. b. combinations of antiretroviral drugs decrease the potential for development of antiretroviral-resistant HIV variants. c. side effects of the drugs are reduced when smaller doses of three different drugs are used rather than large doses of one drug. d. when CD4+ T cell counts are <500/mL, a combination of drugs that have different actions is more effective in slowing HIV growth

*b. combinations of antiretroviral drugs decrease the potential for development of antiretroviral-resistant HIV variants.*

*A patient is admitted with a spinal cord injury at the C7 level. During assessment the nurse identifies the presence of spinal shock on finding?* a. paraplegia with a flaccid paralysis. b. tetraplegia with total sensory loss. c. total hemiplegia with sensory and motor loss. d. spastic tetraplegia with loss of pressure sensation.

*b. tetraplegia with total sensory loss.*

*In caring for a patient after a spinal fusion, which patient symptom would the nurse immediately report to the physician?* a.The patient experiences a single episode of emesis. b.The patient is unable to move the lower extremities. c.The patient is nauseated and has not voided in 6 hours. d.The patient complains of pain at the bone graft donor site.

*b.The patient is unable to move the lower extremities.* After spinal surgery, you should frequently monitor for peripheral neurologic signs. Movement of the arms and legs and assessment of sensation should be unchanged compared with the preoperative status. These assessments are usually repeated every 2 to 4 hours during the first 48 hours after surgery. Compare these findings with the preoperative assessment. Paresthesias, such as numbness and tingling, may not be relieved immediately after surgery. Document any new muscle weakness or paresthesias, and report this information to the surgeon immediately.

*The most common early symptom of a spinal cord tumor is?* a.urinary incontinence. b.back pain that worsens with activity. c.paralysis below the level of involvement. d.impaired sensation of pain, temperature, and light touch.

*b.back pain that worsens with activity.* This is an early symptom STENOSIS too (narrowing of the spinal canal), non-specific back pain that worsens with movement. A tumor would cause stenosis.

*What is the priority action for the nurse to take if the patient with type 2 diabetes complains of blurred vision and irritability?* a. Call the physician. b. Administer insulin as ordered. c. Check the patient's blood glucose level. d. Assess for other neurologic symptoms.

*c. Check the patient's blood glucose level.*

*The treatment for DKA and HHS differs primarily in that?* a. DKA requires administration of bicarbonate to correct acidosis. b. potassium replacement is not necessary in management of HHS. c. HHS requires greater fluid replacement to correct the dehydration. d. administration of glucose is withheld in HHS until the blood glucose reaches a normal level

*c. HHS requires greater fluid replacement to correct the dehydration.*

*The treatment for DKA and HHS differs primarily in that?* a. DKA requires administration of bicarbonate to correct acidosis. b. potassium replacement is not necessary in management of HHS. c. HHS requires greater fluid replacement to correct the dehydration. d. administration of glucose is withheld in HHS until the blood glucose reaches a normal level.

*c. HHS requires greater fluid replacement to correct the dehydration.*

*Ketoacidosis occurs as a complication of diabetes when?* a. illnesses causing nausea and vomiting lead to bicarbonate loss with body fluids. b. the glucose level becomes so high that osmotic diuresis promotes fluid and electrolyte loss. c. an insulin deficit causes the body to metabolize large amounts of fatty acids rather than glucose for energy. d. the patient skips meals after taking insulin, leading to rapid metabolism of glucose and breakdown of fats for energy

*c. an insulin deficit causes the body to metabolize large amounts of fatty acids rather than glucose for energy.*

*Ketoacidosis occurs as a complication of diabetes when?* a. illnesses causing nausea and vomiting lead to bicarbonate loss with body fluids. b. the glucose level becomes so high that osmotic diuresis promotes fluid and electrolyte loss. c. an insulin deficit causes the body to metabolize large amounts of fatty acids rather than glucose for energy. d. the patient skips meals after taking insulin, leading to rapid metabolism of glucose and breakdown of fats for energy

*c. an insulin deficit causes the body to metabolize large amounts of fatty acids rather than glucose for energy.*

Priority Decision: During assessment of a patient with a spinal cord injury, the nurse determines that the patient has a poor cough with diaphragmatic breathing. Based on this finding, the nurse's first action should be to? a. institute frequent turning and repositioning. b. use tracheal suctioning to remove secretions. c. assess lung sounds and respiratory rate and depth. d. prepare the patient for endotracheal intubation and mechanical ventilation

*c. assess lung sounds and respiratory rate and depth.*

*The "rapid" HIV antibody testing is performed on a patient at high risk for HIV infection. The nurse explains that?* a. the test measures the activity of the HIV and reports viral loads as real numbers. b. this test is highly reliable, and in 20 minutes the patient will know if HIV infection is present. c. if the results are positive, another blood test and a return appointment for results will be necessary. d. this test detects drug-resistant viral mutations that are present in viral genes to evaluate resistance to antiretroviral d

*c. if the results are positive, another blood test and a return appointment for results will be necessary.*

An initial incomplete spinal cord injury often results in complete cord damage because of? a. edematous compression of the cord above the level of the injury. b. continued trauma to the cord resulting from damage to stabilizing ligaments. c. infarction and necrosis of the cord caused by edema, hemorrhage, and metabolites. d. mechanical transection of the cord by sharp vertebral bone fragments after the initial injury.

*c. infarction and necrosis of the cord caused by edema, hemorrhage, and metabolites.*

*Before repositioning the patient to the side after a lumbar laminectomy, the nurse?* a. raises the head of the bed 30 degrees. b. has the patient flex the knees and hips. c. places a pillow between the patient's legs. d. has the patient grasp the side rail on the opposite side of the be

*c. places a pillow between the patient's legs.* LOG-ROLL

*A diabetic patient has a serum glucose level of 824 mg/dL (45.7 mmol/L) and is unresponsive. Following assessment of the patient, the nurse suspects diabetic ketoacidosis rather than hyperosmolar hyperglycemic syndrome based on the finding of?* a.polyuria. b.severe dehydration. c.rapid, deep respirations. d.decreased serum potassium.

*c. rapid, deep respirations.*

*A patient with a spinal cord injury has spinal shock. The nurse plans care for the patient based on the knowledge that?* a. rehabilitation measures cannot be initiated until spinal shock has resolved. b. the patient will need continuous monitoring for hypotension, tachycardia, and hypoxemia. c. resolution of spinal shock is manifested by spasticity, hyperreflexia, and reflex emptying of the bladder. d. the patient will have complete loss of motor and sensory functions below the level of the injury, but autonomic functions are not affected.

*c. resolution of spinal shock is manifested by spasticity, hyperreflexia, and reflex emptying of the bladder.*

*Without surgical stabilization, immobilization and traction of the patient with a cervical spinal cord injury most frequently requires the use of?* a. sternal-occipital-mandibular immobilizer (SOMI) brace. b. kinetic beds. c. hard cervical collars. d. skeletal traction with skull tongs.

*c. skeletal traction with skull tongs.* Braces are named after the joint they protect. In lecture the SOMI brace was used on a person standing up. It is a cervical brace but immobilizes it.

*One of the most significant factors in determining when to start antiretroviral therapy in a patient with HIV infection is?* a. whether the patient has high levels of HIV antibodies. b. the confirmation that the patient has contracted HIV infection. c. the patient's readiness to commit to a complex, life-long, uncomfortable drug regimen. d. whether the patient has a support system to help manage the costs and side effects of the drugs.

*c. the patient's readiness to commit to a complex, life-long, uncomfortable drug regimen.*

*Of the following, which is the most appropriate nursing intervention to help an HIV-infected patient adhere to a treatment regimen?* a. "Set up" a drug pillbox for the patient every week. b. Give the patient a videotape and a brochure to view and read at home. c. Tell the patient that the side effects of the drugs are bad but that they go away after a while. d. Assess the patient's routines and find adherence cues that fit into the patient's life circumstances.

*d. Assess the patient's routines and find adherence cues that fit into the patient's life circumstances.*

*When comparing the pathophysiology of type 1 and type 2 diabetes, which statement would be correct for a patient with type 2 diabetes who was admitted to the hospital with pneumonia?* a.The patient must receive insulin therapy to prevent the development of ketoacidosis. b.The patient has islet cell antibodies that have destroyed the ability of the pancreas to produce insulin. c.The patient has minimal or absent endogenous insulin secretion and requires daily insulin injections. d.The patient may have sufficient endogenous insulin to prevent ketosis but is at risk for development of hyperosmolar hyperglycemic syndrome.

*d. The patient may have sufficient endogenous insulin to prevent ketosis but is at risk for development of hyperosmolar hyperglycemic syndrome.*

*A diabetic patient is found unconscious at home, and a family member calls the clinic. After determining that no glucometer is available, the nurse advises the family member to?* a. try to arouse the patient to drink some orange juice. b. administer 10 U of regular insulin subcutaneously. c. call for an ambulance to transport the patient to a medical facility. d. administer glucagon 1 mg intramuscularly (IM) or subcutaneously.

*d. administer glucagon 1 mg intramuscularly (IM) or subcutaneously.*

*Transmission of HIV from an infected individual to another most commonly occurs as a result of?* a.perinatal transmission. b.sharing drug-using equipment. c.low levels of virus in the blood. d.anal or vaginal sexual intercourse.

*d. anal or vaginal sexual intercourse.*

*One indication for surgical therapy of the patient with a spinal cord injury is when?* a. there is incomplete cord lesion involvement. b. the ligaments that support the spine are torn. c. a high cervical injury causes loss of respiratory function. d. evidence of continued compression of the cord is apparent.

*d. evidence of continued compression of the cord is apparent.*

The home care nurse should intervene to correct a patient whose insulin administration includes? a. warming a prefilled refrigerated syringe in the hands before administration. b. storing syringes prefilled with NPH and regular insulin needle-up in the refrigerator. c. placing the insulin bottle currently in use in a small container on the bathroom countertop. d. mixing an evening dose of regular insulin with insulin glargine in one syringe for administration

*d. mixing an evening dose of regular insulin with insulin glargine in one syringe for administration*

*Following a T2 spinal cord injury, the patient develops paralytic ileus. While this condition is present, the nurse anticipates that the patient will need?* a. IV fluids. b. tube feedings. c. parenteral nutrition. d. nasogastric suctioning.

*d. nasogastric suctioning.*

The nurse's primary responsibility for a patient with a suspected disk herniation who has severe pain and muscle spasms is? a.teaching exercises such as straight-leg raises to decrease pain. b.positioning the patient on the abdomen with the legs extended. c.encouraging spine strengthening exercises and taking pain medication as needed. d.assisting the patient to maintain activity restrictions with a gradual increase in activity.

*d.assisting the patient to maintain activity restrictions with a gradual increase in activity.* The overall goals for a patient with acute low back pain (related to disk herniation) are (1) satisfactory pain relief, (2) avoidance of constipation caused by medication and immobility, (3) learning back-sparing practices, and (4) returning to a previous level of activity within prescribed restrictions. SPINAL INJURY THOUGH EARLY MOBILITY IS KEY

*A patient is admitted to the hospital with a C4 spinal cord injury after a motorcycle collision. The patient's BP is 83/49 mm Hg, and his pulse is 39 beats/min, and he remains orally intubated. The nurse identifies this pathophysiologic response as caused by?* a.increased vasomotor tone after injury. b.a temporary loss of sensation and flaccid paralysis below the level of injury. c.loss of parasympathetic nervous system innervation resulting in vasoconstriction. d.loss of sympathetic nervous system innervation resulting in peripheral vasodilation.

*d.loss of sympathetic nervous system innervation resulting in peripheral vasodilation.* Neurogenic shock results from loss of vasomotor tone caused by injury, and it is characterized by hypotension and bradycardia. Loss of sympathetic nervous system innervation causes peripheral vasodilation, venous pooling, and a decreased cardiac output. These effects usually are associated with a cervical or high thoracic injury (T6 or higher).

*Prazosin (Minipress) is an alpha-1 antagonist. What is a common side effect?*

*orthostatic hypotension*

Where should an unopened insulin vial or insulin pen be stored? - How long can a insulin vial or insulin pen be used after is has been opened? - The pen or vial in current use should be kept A. at room temperature B. in the refrigerator C. in the freezer D. in the direct sunlight

-Before it is opened in the refrigerator -28 days after opening -Keep at room temperature after opening

What are the major side effects of Metformin?

-GI disturbances (maybe this si from the MOA on gut absorption of glucose) -metallic taste in the mouth -mild anorexia, nausea, abdominal discomfort, diarrhea

Indicate the lowest level of acute spinal cord injury at which the following effects occur? 1. GI hypomotility with paralytic ileus and gastric distention 2. Loss of all respiratory muscle function 3. Respiratory diaphragmatic breathing 4. Decreased response of the sympathetic nervous system options: Above T5, Above T6, below C4, above C4

1. GI hypomotility with paralytic ileus and gastric distention->above T5 2. Loss of all respiratory muscle function->above C4 3. Respiratory diaphragmatic breathing->below C4 4. Decreased response of the sympathetic nervous system->above T6

In a newly diganosed patient with DM2, what is the first step in treating the disease?

1. Metformin 2. Lifestyle changes

*When a patient is hypoglycemic you should reassess the blood glucose every 15 minutes until greater than _____ ?*

100

A patient is just admitted to the hospital following a spinal cord injury at the level of T4. A priority of nursing care for the patient is monitoring for? 1. return of reflexes. 2. bradycardia with hypoxemia. 3. effects of sensory deprivation. 4. fluctuations in body temperature.

2. bradycardia with hypoxemia. (loss of sympathetic nervous system function causes HR and BP to drop, decreasing perfusion and oxygenation of the blood)

A young adult is hospitalized after an accident that resulted in a complete transection of the spinal cord at the level of C7. The nurse informs the patient that after rehabilitation, the level of function that is most likely to occur is the ability to: 1.breathe with respiratory support. 2.drive a vehicle with hand controls. 3.ambulate with long-leg braces and crutches. 4.use a powered device to handle eating utensils.

2. drive a vehicle with hand controls. Rationale: A pt with injury at the level of C7 to C8 may have the following rehabilitation potential: *ability to transfer self to wheelchair; roll over and sit up in bed; push self on most surfaces; perform most self-care; use wheelchair independently; and drive a car with powered hand controls* (in some pts); attendant care 0 to 6 hours/day.

A patient is just admitted to the hospital following a spinal cord injury at the level of T4. A priority of nursing care is monitoring for which of the following? 1.Return of reflexes 2.Bradycardia with hypoxemia 3.Effects of sensory deprivation 4.Fluctuations in body temperature

2.Bradycardia with hypoxemia Rationale: Neurogenic shock is due to loss of vasomotor tone caused by injury and is characterized by hypotension and bradycardia, which are important clinical clues. Loss of sympathetic nervous system innervation causes peripheral vasodilation, venous pooling, and decreased cardiac output. These effects are generally associated with a cervical or high thoracic injury (T6 or higher). Injury or fracture below the level of C4 results in diaphragmatic breathing if the phrenic nerve is functioning. Even if the injury is below C4, spinal cord edema and hemorrhage can affect the function of the phrenic nerve and cause respiratory insufficiency. Hypoventilation almost always occurs with diaphragmatic respirations because of the decrease in vital capacity and tidal volume, which occurs as a result of impairment of the intercostal muscles. Cervical and thoracic injuries cause paralysis of abdominal muscles and often intercostal muscles. Therefore the pt cannot cough effectively enough to remove secretions, leading to atelectasis and pneumonia. An artificial airway provides direct access for pathogens, making bronchial hygiene and chest physiotherapy extremely important to reduce infection. Neurogenic pulmonary edema may occur secondary to a dramatic increase in sympathetic nervous system activity at the time of injury, which shunts blood to the lungs. In addition, pulmonary edema may occur in response to fluid overload.

Antigen/antibody combination tests can find HIV as soon as ___weeks after exposure to the virus?

3

During assessment of a patient with a spinal cord injury at the level of T2 at the rehabilitation center, which finding would concern the nurse the most? 1.A heart rate of 92 2.A reddened area over the pt's coccyx 3.Marked perspiration on the pt's face and arms 4.A light inspiratory wheeze on auscultation of the lungs

3.Marked perspiration on the pt's face and arms Rationale: Autonomic dysreflexia is a massive uncompensated cardiovascular reaction mediated by the sympathetic nervous system. It occurs in response to visceral stimulation once spinal shock is resolved in pts with spinal cord lesions. The condition is a life-threatening situation that requires immediate resolution. If resolution does not occur, this condition can lead to status epilepticus, stroke, myocardial infarction, and even death. Manifestations include hypertension (up to 300 mm Hg systolic), throbbing headache, marked diaphoresis above the level of the lesion, bradycardia (30 to 40 beats/min), piloerection (erection of body hair) as a result of pilomotor spasm, flushing of the skin above the level of the lesion, blurred vision or spots in the visual fields, nasal congestion, anxiety, and nausea.

Post-Exposure Prophylaxis (PEP) must be started within ____ hours and taken for ____ days?

72 hours, 28 days

*You are caring for a patient with newly diagnosed type 1 diabetes. What information is essential to include in your patient teaching before discharge from the hospital (select all that apply)?* a.Insulin administration b.Elimination of sugar from diet c.Need to reduce physical activity d.Hypoglycemia prevention, symptoms, and treatment

A and D a.Insulin administration d.Hypoglycemia prevention, symptoms, and treatment

Hypoglycemia unawareness

A condition in which a person with diabetes doesn't experience the usual warning symptoms of hypoglycemia. Happens more as someone is diabetic longer. This also happens when someone is on beta-blocker.

*A nurse is caring for a client who is suspected of having HIV. Which of the following diagnostic tests and laboratory values are used to confirm HIV infection? (Select all that apply.)* A. HIV-1/HIV-2 antibody differentiation immunoassay B. Antigen/antibody combination immunoassay C. CD4+ T-lymphocyte count D. CD4+ T-lymphocyte per centage of total lymphocytes E. Cerebrospinal fluid (CSF) analysis

A, B, Others are to stage it

*A nurse is reviewing the health record of a client who has hyperglycemic-hyperosmolar state (HHS). Which of the following data confirms this diagnosis? (Select all that apply.)* A. Evidence of recent myocardial infraction B. BUN 35 mg/dL C. Takes a calcium channel blocker D. Age 77 years E. No insulin production

A, B, C, D NOT E-There is residual insulin production with HHS

A patient is admitted with diabetes mellitus, malnutrition, and cellulitis. The patient's potassium level is 5.6 mEq/L. Which factors could contribute to this laboratory result (select all that apply)? A. The level may be increased as a result of dehydration that accompanies hyperglycemia. B. The patient may be excreting extra sodium and retaining potassium because of malnutrition. C. The level is consistent with renal insufficiency that can develop with renal nephropathy. D. The level may be raised because of metabolic ketoacidosis caused by hyperglycemia. E. The level may be raised because excess insulin is being given and causing potassium shifts

A, C, D

A patient is admitted with diabetes mellitus, malnutrition, and cellulitis. The patient's potassium level is 5.6 mEq/L. Which factors could contribute to this laboratory result (select all that apply)? A. The level may be increased as a result of dehydration that accompanies hyperglycemia. B. The patient may be excreting extra sodium and retaining potassium because of malnutrition. C. The level is consistent with renal insufficiency that can develop with renal nephropathy. D. The level may be raised because of metabolic ketoacidosis caused by hyperglycemia. E. The level may be raised because excess insulin is being given and causing potassium shifts.

A, C, D

You are caring for a patient admitted with diabetes mellitus, malnutrition, and massive gastrointestinal bleed. In analyzing the morning laboratory results, you understand that a potassium level of 5.5 mEq/L could be caused by which factors in this patient (select all that apply)? A. The potassium level may be increased if the patient has renal nephropathy. B. The patient may be excreting extra sodium and retaining potassium because of malnutrition. C. The potassium level may be increased as a result of dehydration that accompanies high blood glucose levels. D. Excess potassium is being released into the blood after a massive transfusion of stored hemolyzed blood.

A, C, D Hyperkalemia may result from hyperglycemia, renal insufficiency, or cell death. Diabetes mellitus, along with the stress of hospitalization and illness, can lead to hyperglycemia. Renal insufficiency is a complication of diabetes. Malnutrition does not cause sodium excretion accompanied by potassium retention; it is not a contributing factor to this patient's potassium level. Stored hemolyzed blood can cause hyperkalemia when large amounts are transfused rapidly. RENAL NEPHROPATHY-UNABLE TO SECRETE K+ DEHYDRATION-more concentrated K+

Indicate whether the following characteristics are associated with hypoglycemia (1), hyperglycemia (2), or both (3)? a. Slurred speech and irritability b. Headache c. Nausea and vomiting d. Too much exercise without food e. Increased dietary intake f. Cold, clammy skin g. Precipitated by stress h. Changes in vision

A-hypoglycemia B-both C-hyperglcyemia D-hypoglycemia E-hyperglycemia F-Hypoglycemia G-hyperglycemia H-both

*A nurse is planning care for a client who suffered a spinal cord injury (SCI) involving a T12 fracture 1 week ago. The client has no muscle control of the lower limbs, bowel, or bladder. Which of the following should be the nurse's highest priority?* A. Prevention of further damage to the spinal cord B. Prevention of contractures of the lower extremities C. Prevention of skin breakdown of areas that lack sensation D. Prevention of postural hypotension when placing the client in a wheelchair

A. The greatest risk to the client during the acute phase of a SCI is further damage to the spinal cord.

*Which instruction should the nurse provide when teaching a patient to mix regular insulin and NPH insulin in the same syringe?* A. "Draw up the clear regular insulin first, followed by the cloudy NPH insulin." B. "It is not necessary to rotate the NPH insulin vial when it is mixed with regular insulin." C. "The order of drawing up insulin does not matter as long as the insulin is refrigerated." D. "Rotate subcutaneous injection sites each day among the arm, thigh, and abdomen."

A. "Draw up the clear regular insulin first, followed by the cloudy NPH insulin." To ensure a consistent response, only NPH insulin is appropriate for mixing with a short-acting insulin. Unopened vials of insulin should be refrigerated; current vials can be kept at room temperature for up to 1 month. Drawing up the regular insulin into the syringe first prevents accidental mixture of NPH insulin into the vial of regular insulin, which could alter the pharmacokinetics of subsequent doses taken out of the regular insulin vial. NPH insulin is a cloudy solution, and it should always be rotated gently to disperse the particles evenly before loading the syringe. Subcutaneous injections should be made using one region of the body (e.g., the abdomen or thigh) and rotated within that region for 1 month.

When teaching a patient infected with HIV regarding transmission of the virus to others, which of the following statements made by the patient would identify a need for further education? A. "I will need to isolate any tissues I use so as not to infect my family." B. "I will notify all of my sexual partners so they can get tested for HIV." C. "Unprotected sexual contact is the most common mode of transmission." D. "I do not need to worry about spreading this virus to others by sweating at the gym."

A. "I will need to isolate any tissues I use so as not to infect my family." HIV is not spread casually. The virus cannot be transmitted through hugging, dry kissing, shaking hands, sharing eating utensils, using toilet seats, or attending school with an HIV-infected person. It is not transmitted through tears, saliva, urine, emesis, sputum, feces, or sweat.

A teaching plan for a patient who is taking lispro (Humalog) should include which instruction by the nurse? A. "Inject this insulin with your first bite of food, because it is very fast acting." B. "The duration of action for this insulin is about 8 to 10 hours, so you'll need a snack." C. "This insulin needs to be mixed with regular insulin to enhance the effects." D. "To achieve tight glycemic control, this is the only type of insulin you'll need."

A. "Inject this insulin with your first bite of food, because it is very fast acting." Lispro is a rapid-acting insulin and has an onset of action of 15 to 30 minutes with a peak action of about 2 hours, not 8 to 10 hours. Because of its rapid onset, it is administered immediately before a meal or with meals to control the blood glucose rise after meals. Lispro insulin must be combined with an intermediate- or a long-acting insulin, not regular insulin (which also is a short-duration insulin), for glucose control between meals and at night. To achieve tight glycemic control, patients must combine different types of insulin based on their duration of action.

Standard precautions should be used when providing care for? A. All patients regardless of diagnosis. B. Pediatric and gerontologic patients. C. Patients who are immunocompromised. D. Patients with a history of infectious diseases.

A. All patients regardless of diagnosis. Standard precautions are designed for all care of all patients in hospitals and health care facilities.

The elderly patient with type 2 diabetes mellitus presents to the clinic with a fever and productive cough. The diagnosis of pneumonia is made. You notice tenting skin, deep tongue furrows, and vital signs of 110/80 mm Hg, 120 beats/minute, and 24 breaths/minute. What assessment is important for you to obtain? A. Blood glucose B. Orthostatic blood pressures C. Urine ketones D. Temperature

A. Blood glucose

A patient is scheduled to start taking insulin glargine (Lantus). On the care plan, a nurse should include which of these outcomes related to the therapeutic effects of the medication? A. Blood glucose control for 24 hours B. Mealtime coverage of blood glucose C. Less frequent blood glucose monitoring D. Peak effect achieved in 2 to 4 hours

A. Blood glucose control for 24 hours Insulin glargine is administered as a once-daily subcutaneous injection for patients with type 1 diabetes. It is used for basal insulin coverage, not mealtime coverage. It has a prolonged duration, up to 24 hours, with no peaks. Blood glucose monitoring is still an essential component to achieve tight glycemic control.

*Which is most important to respond to in a patient presenting with a T3 spinal injury?* A. Blood pressure of 88/60 mm Hg, pulse of 56 beats/minute B. Deep tendon reflexes of 1+, muscle strength of 1+ C. Pain rated at 9 D. Warm, dry skin

A. Blood pressure of 88/60 mm Hg, pulse of 56 beats/minute Neurogenic shock is a loss of vasomotor tone caused by injury, and it is characterized by hypotension and bradycardia. The loss of sympathetic nervous system innervations causes peripheral vasodilation, venous pooling, and a decreased cardiac output. The other options can be expected findings and are not as significant. Patients in neurogenic shock have pink and dry skin, instead of cold and clammy, but this sign is not as important as the vital signs.

Which clinical manifestation do you interpret as representing neurogenic shock in a patient with acute spinal cord injury? A. Bradycardia B. Hypertension C. Neurogenic spasticity D. Bounding pedal pulses

A. Bradycardia

Which clinical manifestation do you interpret as representing neurogenic shock in a patient with acute spinal cord injury? A. Bradycardia B. Hypertension C. Neurogenic spasticity D. Bounding pedal pulses

A. Bradycardia Neurogenic shock results from loss of vasomotor tone caused by injury and is characterized by hypotension and bradycardia. Loss of sympathetic innervation causes peripheral vasodilation, venous pooling, and a decreased cardiac output.

A client is prescribed metoprolol (Lopressor) to treat hypertension. It is important for the nurse to monitor the client for which condition? A. Bradycardia B. Hypertension C. Ankle edema D. Decreased respirations

A. Bradycardia This is a beta-1 receptor blocker. Beta 1=1 heart.

Which of the following clinical manifestations would the nurse interpret as representing neurogenic shock in a patient with acute spinal cord injury? A. Bradycardia B. Hypertension C. Neurogenic spasticity D. Bounding pedal pulses

A. Bradycardia Neurogenic shock is due to the loss of vasomotor tone caused by injury and is characterized by HYPOTENSION and BRADYCARDIA. Loss of sympathetic innervation causes peripheral vasodilation, venous pooling, and a decreased cardiac output.

Before administering metformin (Glucophage), the nurse should notify the prescriber about which laboratory value? A. Creatinine (Cr) level of 2.1 mg/dL B. Hemoglobin (Hgb) level of 9.5 gm/dL C. Sodium (Na) level of 131 mEq/dL D. Platelet count of 120,000/mm3

A. Creatinine (Cr) level of 2.1 mg/dL Metformin can reach toxic levels in individuals with renal impairment, which is indicated by a rise in the serum creatinine level. The prescriber may need to be notified of the hemoglobin, sodium, and platelet values, but they would not affect the administration of metformin.

Before administering metformin (Glucophage), the nurse should notify the prescriber about which laboratory value? A. Creatinine (Cr) level of 2.1 mg/dL B. Hemoglobin (Hgb) level of 9.5 gm/dL C. Sodium (Na) level of 131 mEq/dL D. Platelet count of 120,000/mm3

A. Creatinine (Cr) level of 2.1 mg/dL *KIDNEY MUST BE ABLE TO ABSORB-IN MET-FOR-MIN (Really kidney excretes and is just important to blood levels I think but its catchy) Metformin can reach toxic levels in individuals with renal impairment, which is indicated by a rise in the serum creatinine level. The prescriber may need to be notified of the hemoglobin, sodium, and platelet values, but they would not affect the administration of metformin.

The nurse is preparing to administer insulin to a patient who has been prescribed a conventional therapy insulin regimen. What is the best action by the nurse? A. Draw up the Regular insulin (Humulin R) first. B. Draw up the NPH insulin (Humulin N) first. C. Prepare two separate insulin injections for the patient.

A. Draw up the Regular insulin (Humulin R) first. (inject air into NPH, inject air into Regular and draw up, draw up NPH last and inject both) *Think R is fi-R-st

The nurse is evaluating a 45-year-old patient diagnosed with type 2 diabetes mellitus. Which of the following symptoms reported by the patient is considered one of the classic clinical manifestations of diabetes? A. Excessive thirst B. Gradual weight gain C. Overwhelming fatigue D. Recurrent blurred vision

A. Excessive thirst The classic symptoms of diabetes are polydipsia (excessive thirst), polyuria, (excessive urine output), and polyphagia (increased hunger).

Laboratory results are available for a 54-year-old patient with a 15-year history of diabetes. Which result follows the expected pattern accompanying macrovascular disease as a complication of diabetes? A. Increased triglyceride levels B. Decreased low-density lipoprotein levels C. Increased high-density lipoprotein levels D. Decreased very-low-density lipoprotein levels

A. Increased triglyceride levels Macrovascular complications of diabetes include changes in medium- and large-sized blood vessels. They include cerebrovascular, cardiovascular, and peripheral vascular disease. Increased triglyceride levels are associated with these macrovascular changes. For this reason, the patient should limit the amount of fat in the diet.

The nurse is preparing to administer a dose of clonidine (Catapres). Which is the best description of the action of this drug? A. It selectively activates alpha2 receptors in the central nervous system (CNS). B. It causes peripheral activation of alpha1 and alpha2 receptors. C. It depletes sympathetic neurons of norepinephrine. D. It directly blocks alpha and beta receptors in the periphery.

A. It selectively activates alpha2 receptors in the central nervous system (CNS). Clonidine is an alpha 2-adrenergic agonist that causes SELECTIVE activation of alpha 2 receptors in the CNS. This in turn reduces sympathetic outflow to the blood vessels and the heart. Although the body's responses are similar to those from a peripheral adrenergic blocker, clonidine's action occurs in the CNS.

*The nurse is caring for a pregnant patient who is experiencing a new episode of hypertension. Which agent does the nurse anticipate will be prescribed for this patient?* A. Methyldopa (Aldomet) B. Propranolol (Inderal) C. Captopril (Capoten) D. Valsartan (Diovan)

A. Methyldopa (Aldomet) This is an Alpha-2 agonist that acts to inhibit NE, Hypertension is the most common complication of pregnancy, occurring in about 10% of pregnant patients. When drug therapy is initiated during pregnancy, methyldopa is the traditional agent of choice because of its limited effects on the fetus. Patients with pre-existing hypertension typically can continue taking the antihypertensives they previously were prescribed except for angiotensin-converting enzyme (ACE) inhibitors, such as captopril; angiotensin II receptor blockers (ARBs), such as valsartan; and direct renin inhibitors, such as aliskiren.

The nurse is caring for a patient receiving inhalant anesthesia and succinylcholine (Anectine). The patient develops muscle rigidity and a temperature of 105°F. What nursing action is essential? A. Prepare to administer dantrolene (Dantrium) B. Administer normal saline at 50 mL/hr C. Administer morphine sulfate to relieve pain D. Prepare for intubation and mechanical ventilation

A. Prepare to administer dantrolene (Dantrium) The patient is showing symptoms of malignant hyperthermia, a rare and potentially fatal complication of succinylcholine and inhalation anesthesia. Treatment includes discontinuation of succinylcholine and anesthesia; application of cooling blankets and ice packs; and IV administration of dantrolene, a muscle relaxant.

Insulin Pens and Vials in current use should be kept at? A. Room Temp B. Fridge C. Freezer D. Direct Sunlight

A. Room Temp

Hyporeflexia, loss of sensation and flaccid paralysis are associated with? A. Spinal Shock B. Neurogenic shock

A. Spinal Shock

Which statement is true regarding the difference between type 1 and type 2 diabetes mellitus? A. Type 2 diabetes has decreased insulin secretion and increased insulin resistance. B. Type 2 diabetes has a total dependency on an outside source of insulin. C. Type 1 diabetes typically occurs in older, obese adults. D. Type 1 diabetes can result in hyperosmolar hyperglycemic syndrome (HHS).

A. Type 2 diabetes has decreased insulin secretion and increased insulin resistance.

Which intervention should you perform in the acute care of a patient with autonomic dysreflexia? A. Urinary catheterization B. Administration of benzodiazepines C. Suctioning of the patient's upper airway D. Placement of the patient in the Trendelenburg position

A. Urinary catheterization

Which of the following interventions should the nurse perform in the acute care of a patient with autonomic dysreflexia? A. Urinary catheterization B. Administration of benzodiazepines C. Suctioning of the patient's upper airway D. Placement of the patient in the Trendelenburg position

A. Urinary catheterization Because the most common cause of autonomic dysreflexia is bladder irritation, immediate catheterization to relieve bladder distention may be necessary. The patient should be positioned upright. Benzodiazepines are contraindicated and suctioning is likely unnecessary.

The priority nursing intervention for the patient with myasthenia gravis who is receiving pyridostigmine (Mestinon) includes? A. timing drug administration so that chewing and swallowing are enhanced. B. withholding the drug if muscle function improves. C. assessing for constipation and paralytic ileus. D. monitoring renal and hepatic function.

A. timing drug administration so that chewing and swallowing are enhanced.

You would correctly identify that the most common early symptom(s) of myasthenia gravis are A. weakness, fatigue, and ptosis. B. significant unilateral weakness. C. nausea, dizziness, and dysphagia. D. numbness and tingling of the extremities.

A. weakness, fatigue, and ptosis.

*Prazosin (Minipress) is a _______ ______ that is used for hypertension and causes orthostatic hypotension as a common side effect?*

Alpha 1 Antagonist

Propranolol is contraindicated for ________ ?

Asthma (non-selective so it blocks both Beta-1 and Beta-2) Beta-2 is lungs so it constricts bronchioles AND diabetes (Beta 2 blockade inhibits glycogyenolysis, causing hypoglycemia and preventing awareness by blokcing early symptoms.

_____________ is related to reflex stimulation of the sympathetic nervous system, which is reflected by hypertension (up to 300 mm Hg), bradycardia, throbbing headache, and diaphoresis.

Autonomic dysreflexia

A nurse is reviewing laboratory reports of a client who has hyperglycemic-hyperosmolar state (HHS). Which of the following is an expected finding? A. Serum pH 7.2 B. Serum osmolarity 350 mOsm/L C. Serum potassium 3.8 mg/dL D. Serum creatinine 0.8 mg/dL

B A client who has HHS would have a serum osmolarity greater than 320 mOsm/L. Creatnine would be elevated from dehydration causing poor perfusion and prerenal injury. Potassium would be elevated from dehydration. pH would not be acidic no ketosis.

A nurse is assessing a client who has diabetic ketoacidosis and ketones in the urine. Which of the following are expected findings? (Select all that apply.) A. Weight gain B. Fruity odor of breath C. Abdominal pain D. Kussmaul respirations E. Metabolic acidosis

B, C, D, E NOT A-There is weight loss not gain with DKA

A nurse caring for a patient who has diabetic ketoacidosis recognizes which characteristics in the patient? (Select all that apply.) A. Type 2 diabetes B. Altered fat metabolism leading to ketones C. Arterial blood pH of 7.35 to 7.45 D. Sudden onset, triggered by acute illness E. Plasma osmolality of 300 to 320 mOsm/L

B, D, E Diabetic ketoacidosis is the most severe manifestation of insulin deficiency in patients with type 1 diabetes. It develops and worsens acutely over several hours to days. Alterations in fat metabolism lead to the production of ketones and ketoacids. Increased ketoacid levels lead to a fall in arterial blood pH below 7.35. Altered glucose metabolism leads to hyperglycemia, water loss, and an elevated plasma osmolality (285 to 295 mOsm/L).

The nurse has taught a patient admitted with diabetes, cellulitis, and osteomyelitis about the principles of foot care. The nurse evaluates that the patient understands the principles of foot care if the patient makes which of the following statements? A. "I should only walk barefoot in nice dry weather." B. "I should look at the condition of my feet every day." C. "I am lucky my shoes fit so nice and tight because they give me firm support." D. "When I am allowed up out of bed, I should check the shower water with my toes."

B. "I should look at the condition of my feet every day." Patients with diabetes mellitus need to inspect the feet daily for broken areas that are at risk for delayed wound healing. Water temperature should be tested with the hands first. Properly fitted (not tight) shoes should be worn at all times.

Which statement made by the patient indicates the best understanding of teaching related to a new prescription for atenolol (Tenormin)? A. "I will increase my fluids to prevent constipation." B. "I will not stop taking this medication abruptly." C. "I will take the first dose of this medicine at night." D. "I will wear sunscreen and a hat when I work in the sun."

B. "I will not stop taking this medication abruptly." Atenolol is a beta blocker and can cause rebound cardiac excitation if withdrawn abruptly. Patients should carry an adequate supply when traveling. It does not commonly cause constipation, first-dose hypotension, or photosensitivity.

Which statement is the most appropriate to include in the teaching plan for a 30-year-old woman beginning a new prescription of clonidine (Catapres)? A. "This medication often is used to manage hypertension during pregnancy." B. "If you stop taking this drug abruptly, your blood pressure might go up very high." C. "You will need to have your blood drawn regularly to check for anemia." D. "Take this medication first thing in the morning to reduce nighttime wakefulness."

B. "If you stop taking this drug abruptly, your blood pressure might go up very high." *Clonidine is an APHA-2 AGONIST. It is not used for pregnancy but another alpha-2 agonist called Methyldopa is*

A 54-year-old patient admitted with type 2 diabetes, asks the nurse what "type 2" means. Which of the following is the most appropriate response by the nurse? A. "With type 2 diabetes, the body of the pancreas becomes inflamed." B. "With type 2 diabetes, insulin secretion is decreased and insulin resistance is increased." C. "With type 2 diabetes, the patient is totally dependent on an outside source of insulin." D. "With type 2 diabetes, the body produces autoantibodies that destroy b-cells in the pancreas."

B. "With type 2 diabetes, insulin secretion is decreased and insulin resistance is increased."

When should the nurse expect that a patient might experience a hypoglycemic reaction to Regular inulin if it was administered subcutaneously at 07:00? A. 08:00 - 09:00 B. 09:00 - 11:00 C. 11:00 -15:00 D. 15:00 -17:00

B. 09:00 - 11:00

*In 2013 the U.S Preventive Services Task Force made recommendations that HIV screening be performed on all: (circle all that apply)?* A. Adults over 65 B. Adolescents over 15 years of age 15-65 years of age C. Pregnant women D. College students E. Men who have sex with men (MSM)

B. Adolescents over 15 years of age C. Pregnant women D. College students (usually between ages of 15-65) E. Men who have sex with men (MSM)

The patient is managed with NPH and regular insulin injections before breakfast and before dinner. When is the patient most likely to have a hypoglycemic reaction? A. After breakfast B. Before lunch C. During lunch D. After lunch

B. Before lunch

The nurse is caring for a patient newly diagnosed with HIV. The patient asks what would determine the actual development of AIDS. The nurse's response is based on the knowledge that which of the following is a diagnostic criterion for AIDS? A. Presence of HIV antibodies B. CD4+ T cell count <200/µl C. White blood cell count <5000/µl D. Presence of oral hairy leukoplakia

B. CD4+ T cell count <200/µl CD-4-STAGES AIDS. Diagnostic criteria for AIDS include a CD4+ T-cell count <200/µl and/or the development of specified opportunistic infections, cancers, wasting syndrome, or dementia. The other options may be found in patients with HIV disease, but do not define the advancement of the disease to AIDS.

*The nurse provides preoperative teaching for a client on the side effects of atropine given as a preoperative medication. The nurse informs the client to expect:* A. Decreased heart rate. B. Dry mouth. C. Sedation. D. Increased urination.

B. Dry mouth.

The nurse provides preoperative teaching for a client on the side effects of atropine given as a preoperative medication. The nurse informs the client to expect: A. Decreased heart rate. B. Dry mouth. C. Sedation. D. Increased urination.

B. Dry mouth.

erase

B. Frequent and thorough hand washing Hand washing remains the mainstay of the prevention of HAIs. Gloves, masks, and antibiotics may be appropriate in specific circumstances, but none of these replaces the central role of vigilant, thorough hand washing.

The nurse, caring for a client receiving a cholinergic drug, should observe for increased symptoms of which of the following processes? A. Hypertension B. Heart failure C. Cardiac dysrhythmias D. Asthma

B. Heart failure A-hypotention C-this is a side effect of anti-cholinergics D-it does actually

Patients who experience HHS usually have a history of: A. Congestive Heart Failure B. Inadequate Fluid Intake C. Chronic Kidney Disease D. Dementia

B. Inadequate Fluid Intake - Also mental depression, and polyuria

The patient with type 1 diabetes arrives in the emergency department with a glucose level of 390 mg/dL and positive result for ketones. Vital signs are 110/70 mm Hg, 120 beats/minute, and 28 deep, sighing respirations/minute. What is the priority need for the patient? A. Oxygen B. Intravenous (IV) fluids C. Insulin D. Metformin (Glucophage)

B. Intravenous (IV) fluids Administer 0.9% NS at 1000 ml/hr first followed by 0.45% NS. Replace the intravascular volume first then hydrate cells. You want to use insulin in the hospital for better control but replace potassium first to avoid fatal hypokalemia.

The nurse is preparing to give neostigmine (Prostigmin). What best describes the action of this drug? A. It inhibits acetylcholine at all cholinergic synapses. B. It prevents inactivation of acetylcholine. C. It prevents activation of muscarinic receptors. D. It stimulates activation of adrenergic receptors.

B. It prevents inactivation of acetylcholine. Neostigmine is a cholinesterase inhibitor. As such, it prevents the inactivation of acetylcholine, allowing it to linger at the synapses. It lacks selectivity and thus intensifies transmission at all cholinergic junctions.

The nurse is beginning to teach a diabetic patient about vascular complications of diabetes. Which of the following information would be appropriate for the nurse to include? A. Macroangiopathy does not occur in type 1 diabetes but rather in type 2 diabetics who have severe disease. B. Microangiopathy is specific to diabetes and most commonly affects the capillary membranes of the eyes, kidneys, and skin. C. Renal damage resulting from changes in large- and medium-sized blood vessels can be prevented by careful glucose control. D. Macroangiopathy causes slowed gastric emptying and the sexual impotency experienced by a majority of patients with diabetes.

B. Microangiopathy is specific to diabetes and most commonly affects the capillary membranes of the eyes, kidneys, and skin.

Loss of sympathetic nervous system innervation causing peripheral vasodilation, venous pooling and decreased cardiac output. Hypotension and bradycardia are clinical cues? A. Spinal Shock B. Neurogenic shock

B. Neurogenic shock

Which label most aptly describes the drug atropine (Sal-Tropine)? A. Cholinergic B. Parasympatholytic C. Muscarinic agonist D. Parasympathomimetic

B. Parasympatholytic Atropine is an anti-cholinergic.

You are caring for a patient admitted with a spinal cord injury after a motor vehicle accident. The patient exhibits a complete loss of motor, sensory, and reflex activity below the injury level. What is this condition? A. Central cord syndrome B. Spinal shock syndrome C. Anterior cord syndrome D. Brown-Séquard syndrome

B. Spinal shock syndrome About 50% of people with acute spinal cord injury experience a temporary loss of reflexes, sensation, and motor activity that is known as spinal shock. Central cord syndrome is manifested by motor and sensory loss greater in the upper extremities than the lower extremities. Anterior cord syndrome results in motor and sensory loss but not loss of reflexes. Brown-Séquard syndrome is characterized by ipsilateral loss of motor function and contralateral loss of sensory function.

Which of the following drugs causes hypoglycemia and weight gain? A. MetFORMIN (Glucophage) B. Sulfonylureas C. Thiazolidinediones (TZDs or Glitazones) D. GLP-1 agonists

B. Sulfonylureas

A nurse assesses a patient who is taking pramlintide (Symlin) with mealtime insulin. Which finding requires immediate follow-up by the nurse? A. Skin rash B. Sweating C. Itching D. Pedal edema

B. Sweating Pramlintide is a new type of antidiabetic medication used as a supplement to mealtime insulin in patients with type 1 and 2 diabetes. Hypoglycemia, which is manifested by sweating, tremors, and tachycardia, is the adverse reaction of most concern. Skin rash, itching, and edema are not adverse effects of pramlintide.

Which elevated laboratory finding is the best indication of potential diabetic nephropathy? A. Blood urea nitrogen (BUN) level B. Urine albumin-to-creatinine ratio C. Urine specific gravity D. Chloride (Cl-) level

B. Urine albumin-to-creatinine ratio Screening for nephropathy depends on the urinary albumin-to-creatinine ratio and a serum creatinine level. (Creatinine indicates how well the kidney is filtering). BUN alone, without correction to creatinine, can indicate many other issues, including dehydration and liver function. Unless there is renal failure, urine specific gravity is more indicative of dehydration. Chloride is not a direct indicator of kidney function

Insulin absorption is fastest and most consistent in which region? A. upper arm B. abdomen C. thigh D. ventral gluteal (hip)

B. abdomen (all other sites listed are alternate sites)

You would correctly identify the pathophysiologic etiology of myasthenia gravis as a deficit of A. dopamine. B. acetylcholine. C. myelin. D. albumin.

B. acetylcholine.

*Antiretroviral drugs are used to?* A. cure acute HIV infection. B. decrease viral RNA levels. C. treat opportunistic diseases. D. decrease pain and symptoms in terminal disease.

B. decrease viral RNA levels.

A patient is admitted to the intensive care unit (ICU) with a C7 spinal cord injury and diagnosed with Brown-Séquard syndrome. What would you most likely find on physical examination? A. Upper extremity weakness only B. Complete motor and sensory loss below C7 C. Loss of position sense and vibration in both lower extremities D. Ipsilateral motor loss and contralateral sensory loss below C7

Brown-Séquard syndrome is a result of damage to one half of the spinal cord. This syndrome is characterized by a loss of motor function and position and vibratory sense, as well as vasomotor paralysis on the same side (ipsilateral) as the lesion. The opposite (contralateral) side has loss of pain and temperature sensation below the level of the lesion.

*A nurse is preparing to administer IV fluids to a client who has diabetic ketoacidosis. Which of the following is an appropriate nursing action?* A. Administer an IV infusion of regular insulin at 0.3 unit/kg/hr. B. Administer an IV infusion of 0.45% sodium chloride. C. Rapidly administer an IV infusion of 0.9% sodium chloride. D. Add glucose to the IV infusion when serum glucose is 350 mg/dL.

C. The nurse should rapidly administer an IV infusion of 0.9% sodium chloride, an isotonic uid, as prescribed to maintain blood perfusion to vital organs.

Which symptoms reported by a patient with diabetes mellitus are most important to follow-up? A. "My vision has been getting fuzzier over the past year." B. "I cannot read the small print anymore." C. "There is something like a veil of blackness coming across my vision." D. "I have yellow discharge from one eye."

C. "There is something like a veil of blackness coming across my vision."

The nurse is providing care for a patient who has been living with HIV for several years. Which of the following assessment findings most clearly indicates an acute exacerbation of the disease? A. A new onset of polycythemia B. Presence of mononucleosis-like symptoms C. A sharp decrease in the patient's CD4+ count D. A sudden increase in the patient's WBC count

C. A sharp decrease in the patient's CD4+ count A decrease in CD4+ count signals an exacerbation of the severity of HIV. Polycythemia is not characteristic of the course of HIV. A patient's WBC count is very unlikely to suddenly increase, with decreases being typical. Mononucleosis-like symptoms such as malaise, headache, and fatigue are typical of EARLY HIV infection and seroconversion.

The nurse is caring for a group of patients diagnosed with Alzheimer's disease (AD). Which neurotransmitter level is decreased by as much as 90% in patients with severe AD? A. Norepinephrine B. Serotonin C. Acetylcholine D. Dopamine

C. Acetylcholine

The patient has type 1 diabetes mellitus and is found unresponsive with cool and clammy skin. What action is a priority? A. Obtain a serum glucose level. B. Give hard candy under the tongue. C. Administer IM glucagon per standing order. D. Notify the health care provider.

C. Administer IM glucagon per standing order.

A patient who took NPH insulin at 0800 reports feeling weak and tremulous at 1700. Which action should the nurse take? A. Take the patient's blood pressure. B. Give the patient's PRN dose of insulin. C. Check the patient's capillary blood sugar. D. Advise the patient to lie down with the legs elevated.

C. Check the patient's capillary blood sugar. The patient is showing symptoms of hypoglycemia at 5 PM. NPH has a peak action of 8 to 10 hours after administration. Based on the duration of action of NPH insulin, the patient's hypoglycemic symptoms are from the 8 AM injection of NPH insulin. An injection of NPH insulin at 2 AM, 1 PM, or 3 PM would not cause hypoglycemic symptoms based on the average duration of action of NPH insulin.

A patient who took NPH insulin at 0800 reports feeling weak and tremulous at 1700. Which action should the nurse take? A. Take the patient's blood pressure. B. Give the patient's PRN dose of insulin. C. Check the patient's capillary blood sugar. D. Advise the patient to lie down with the legs elevated.

C. Check the patient's capillary blood sugar. The patient is showing symptoms of hypoglycemia at 5 PM. NPH has a peak action of 8 to 10 hours after administration. Based on the duration of action of NPH insulin, the patient's hypoglycemic symptoms are from the 8 AM injection of NPH insulin. An injection of NPH insulin at 2 AM, 1 PM, or 3 PM would not cause hypoglycemic symptoms based on the average duration of action of NPH insulin. Hypoglycemia would occur at the the peak.

*You are admitting a patient who complains of new-onset lower back pain. To differentiate between the pain of a lumbar herniated disk and lower back pain from other causes, what is the best question to ask the patient?* A. Is the pain worse in the morning or in the evening? B. Is the pain sharp or stabbing, or burning or aching? C. Does the pain radiate down the buttock or into the leg? D. Is the pain totally relieved by analgesics, such as acetaminophen (Tylenol)?

C. Does the pain radiate down the buttock or into the leg? Lower back pain associated with herniated lumbar disk is accompanied by pain radiated along the sciatic nerve, and it is commonly described as traveling through the buttock to the posterior thigh or down the leg. The herniated disk compresses spinal nerves as they exit the spinal column. The other questions do not elicit this information.

What is the correct teaching regarding oral antidiabetic medications? A. Double the glipizide (Glucotrol) dose if consuming alcohol. B. Hold all antidiabetic medication if vomiting is related to the flu. C. Hold metformin 48 hours before a procedure with contrast medium. D. Acarbose (Precose) is taken 2 hours after meals.

C. Hold metformin 48 hours before a procedure with contrast medium. *Metformin (Glucophage) is held 24 to 48 hours before and after a procedure with contrast medium to minimize the risk of acute renal failure and lactic acidosis.*

What is the correct teaching regarding oral antidiabetic medications? A. Double the glipizide (Glucotrol) dose if consuming alcohol. B. Hold all antidiabetic medication if vomiting is related to the flu. C. Hold metformin 48 hours before a procedure with contrast medium. D. Acarbose (Precose) is taken 2 hours after meals.

C. Hold metformin 48 hours before a procedure with contrast medium. Renal failure and lactic acidosis can result

A patient is admitted with diabetes mellitus, has a glucose level of 380 mg/dl, and a moderate level of ketones in the urine. As the nurse assesses for signs of ketoacidosis, which of the following respiratory patterns would the nurse expect to find? A. Central apnea B. Hypoventilation C. Kussmaul respirations D. Cheyne-Stokes respirations

C. Kussmaul respirations In diabetic ketoacidosis, the lungs try to compensate for the acidosis by blowing off volatile acids and carbon dioxide. This leads to a pattern of Kussmaul respirations, which are deep and non-labored.

The nurse is teaching a 54-year-old patient with diabetes about proper composition of the daily diet. The nurse explains that the guideline for carbohydrate intake is which of the following? A. 80% of daily intake B. Minimum of 80 g/day C. Minimum of 130 g/day D. Maximum of 130 g/day

C. Minimum of 130 g/day The recommendation for carbohydrate intake is a minimum of 130 g/day. Low-carbohydrate diets are not recommended for diabetes management.

*What is most important action for a patient who has a suspected cervical spinal injury?* A. Apply a soft foam cervical collar. B. Perform a neurologic check. C. Place the patient on a firm surface. D. Assess function of cranial nerves IX and X.

C. Place the patient on a firm surface. A patient with a suspected cervical spine injury should be immobilized with a hard collar and placed on a firm surface. This TAKES PRIORITY over any further assessment. A soft foam collar DOES NOT provide immobilization.

What is most important action for a patient who has a suspected cervical spinal injury? A. Apply a soft foam cervical collar. B. Perform a neurologic check. C. Place the patient on a firm surface. D. Assess function of cranial nerves IX and X.

C. Place the patient on a firm surface. A patient with a suspected cervical spine injury should be immobilized with a hard collar and placed on a firm surface. This takes priority over any further assessment. A soft foam collar does not provide immobilization.

Why does the nurse anticipate administering metoprolol (Lopressor) rather than propranolol (Inderal) for diabetic patients who need a beta-blocking agent? A. Metoprolol is less likely to cause diabetic nephropathy. B. Metoprolol helps prevent retinopathy in individuals with diabetes C. Propranolol causes both beta1 and beta2 blockade. D. Propranolol is associated with a higher incidence of foot ulcers.

C. Propranolol causes both beta1 and beta2 blockade. Propranolol is a non-selective beta-blocker so it blocks beta-2, which causes broncho-constriction and inhibits glycogenolysis. It also blocks tachycardia, an early warning sign of hypoglycemia. Metoprolol is a second-generation beta blocker and as such is more selective. At therapeutic doses, it causes less bronchoconstriction and *less suppression of glycogenolysis*, which can cause problems in diabetic patients. Propranolol blocks both beta1 and beta2 receptors.

*A nurse is teaching clients in an outpatient facility about the use of insulin to treat type 1 diabetes mellitus. For which of the following types of insulin should the nurse tell the clients to expect a peak effect 1 to 5 hr after administration?* A. Insulin glargine (Lantus) B. NPH insulin (Humulin N) C. Regular insulin (Humulin R) D. Insulin lispro (Humalog)

C. Regular insulin (Humulin R)

The patient with diabetes mellitus confides to you that he is afraid he will lose his vision like his friend did. What is the best response? A. Encourage the patient to look at the blessings he has instead of worrying. B. Verify there is nothing that can influence that outcome but new measures to cope are being developed. C. Stress that tight glycemic control can minimize microvascular complications such as retinopathy. D. Remind the patient that every person is different.

C. Stress that tight glycemic control can minimize microvascular complications such as retinopathy.

A patient is taking glipizide (Glucotrol) and a beta-adrenergic medication. A nurse is teaching hypoglycemia awareness and should warn the patient about the absence of which symptom? A. Vomiting B. Muscle cramps C. Tachycardia D. Chills

C. Tachycardia Glipizide is a sulfonylurea oral hypoglycemic medication that acts to promote insulin release from the pancreas. Beta-adrenergic blockers can mask early signs of sympathetic system responses to hypoglycemia; the most important of these is tachycardia, which is the most common adverse effect of glipizide. Vomiting, muscle cramps, and chills are not symptoms of activation of the sympathetic nervous system that arise when glucose levels fall.

*A patient is taking a Sulfonylurea and and a beta-adrenergic medication. A nurse is teaching hypoglycemia awareness and should warn the patient about the absence of which symptom?* A. Vomiting B. Muscle cramps C. Tachycardia D. Chills

C. Tachycardia Sulfonylureas are oral hypoglycemic medication that acts to promote insulin release from the pancreas. They carry a risk of hypoglycemia sonic their MOA is insulin release. Beta-adrenergic blockers can mask early signs of sympathetic system responses to hypoglycemia. Vomiting, muscle cramps, and chills are not symptoms of activation of the sympathetic nervous system that arise when glucose levels fall.

*A patient with a C7 spinal cord injury undergoing rehabilitation tells you he must have the flu because he has a bad headache and nausea. What is your initial action?* A. Call the physician. B. Check the patient's temperature. C. Take the patient's blood pressure. D. Elevate the head of the bed to 90 degrees.

C. Take the patient's blood pressure. Autonomic dysreflexia is a massive, uncompensated cardiovascular reaction mediated by the sympathetic nervous system. Manifestations include hypertension (up to 300 mm Hg systolic), throbbing headache, marked diaphoresis above the level of the lesion, bradycardia (30 to 40 beats/minute), piloerection, flushing of the skin above the level of the lesion, blurred vision or spots in the visual fields, nasal congestion, anxiety, and nausea. It is important to measure blood pressure when a patient with a spinal cord injury complains of a headache. Other nursing interventions in this serious emergency are elevation of the head of the bed 45 degrees or sitting the patient upright, notification of the physician, and assessment to determine the cause (see Table 61-7 for the causes and symptoms of autonomic dysreflexia). You must monitor blood pressure frequently during the episode. An α-adrenergic blocker or an arteriolar vasodilator may be administered.

*A nurse is teaching a client who is beginning highly active antiretroviral therapy (HAART) for HIV infection about ways to prevent medication resistance. Which of the following should the nurse teach the client about resistance?* A. Taking low dosages of antiretroviral medication minimizes resistance. B. Taking one antiretroviral medication at a time minimizes resistance. C. Taking medication at the same times daily without skipping doses minimizes resistance. D. Changing the medication regimen when adverse effects occur minimizes resistance.

C. Taking medication at the same times daily without skipping doses minimizes resistance.

Which of the following drugs causes water retention and should be avoided in patients with heart failure? A. MetFORMIN (Glucophage) B. Sulfonylureas C. Thiazolidinediones (TZDs or Glitazones) D. GLP-1 agonists

C. Thiazolidinediones (TZDs or Glitazones)

Which is a sign of hyperglycemia? A. Tremors B. Cold, clammy skin C. increased urination D. increased blood pressure

C. increased urination

C3-C5 is the phrenic nerve, which innervates the diaphragm. An injury above ___ causes total loss of all respiratory function while an injury below ___ allows for diaphragmatic breathing?

C4-think C4 dia-4-matiC breathing. Both can be ineffective for oxygenation still.

DUMBBELLS IS A MNEMONIC FOR EXCESSIVE CHOLINERGIC SYMPTOMS. WHAT DOES DUMBBELLS STAND FOR?

D-Diahrrea U-Urination M-Miny pupils (Miosis) B-Bradycardia B-Brochospasm (difficulty breathing) constriction E-Emesis L-Lacrimation L-Lethargy S-Salivation

A nurse is completing discharge instructions with a client who has AIDS. Which of the following statements by the client indicates an understanding of the teaching? A. "I will wear gloves while changing the pet litter box." B. "I will rinse raw fruits with water before eating them." C. "I will wear a mask when around family members who are ill." D. "I will cook vegetables before eating them."

D. "I will cook vegetables before eating them."

A 54-year-old patient admitted with type 2 diabetes, asks the nurse what "type 2" means. Which of the following is the most appropriate response by the nurse? A. "With type 2 diabetes, the body of the pancreas becomes inflamed." B. "With type 2 diabetes, insulin secretion is decreased and insulin resistance is increased." Correct C. "With type 2 diabetes, the patient is totally dependent on an outside source of insulin." D. "With type 2 diabetes, the body produces autoantibodies that destroy b-cells in the pancreas."

D. "With type 2 diabetes, insulin secretion is decreased and insulin resistance is increased."

Anti-diabetic medications for Type 2 diabetes act by? A. Decreasing insulin resistance B. Increasing insulin production C. Decreasing hepatic glucose production D. All of the above

D. All of the above

A nurse prepares to administer a new prescription for bethanechol (Urecholine). Which information in the patient's history should prompt the nurse to consult with the prescriber before giving the drug? A. Constipation B. Hypertension C. Psoriasis D. Asthma

D. Asthma Muscarinic agonists induce bronchospasm, which would cause problems for a patient with a history of asthma.

*A nurse admits a client with newly diagnosed diabetes mellitus. When reviewing the client's laboratory work, the result consistent with this diagnosis is?* A. Blood glucose of 30 mg/dL B. Negative urine acetone C. Blood pH of 7.38 D. Bicarbonate level of 12 mEq/L

D. Bicarbonate level of 12 mEq/L ROME-respiratory opposite, metabolic equal Metabolic acidosis->low pH and low bicarb

The nurse is caring for a patient receiving methyldopa (Aldomet). Which laboratory test is the most important to obtain before treatment? A. Creatinine level B. Serum glucose level C. White blood cell count D. Hematocrit

D. Hematocrit This APLHPA-2 AGONIST is an antihypertensive used during pregnancy. A few of those patients develop hemolytic anemia. Coombs' test and blood cell counts (hematocrit, hemoglobin, or red cell count) should be done to establish a baseline and periodically throughout long-term treatment.

A nurse is caring for a client who is taking ritonavir (Norvir), a protease inhibitor, to treat HIV infection. For which of the following abnormalities in laboratory values should the nurse monitor? A. Increased TSH and T4 levels B. Decreased ALT and AST levels C. Hypoglycemia D. Hyperlipidemia

D. Hyperlipidemia

A client is given epinephrine (Adrenalin), an adrenergic agonist (sympathomimetic). The nurse should monitor the client for which condition? A. Decreased pulse B. Pupil constriction C. Bronchial constriction D. Increased blood pressure

D. Increased blood pressure

*When planning care for a patient with a C5 spinal cord injury, which nursing diagnosis is the highest priority?* A. Impaired tissue integrity due to paralysis B. Impaired urinary elimination due to quadriplegia C. Ineffective coping due to the extent of trauma D. Ineffective airway clearance due to high cervical spinal cord injury

D. Ineffective airway clearance due to high cervical spinal cord injury

When planning care for a patient with a C5 spinal cord injury, which nursing diagnosis is the highest priority? A. Impaired tissue integrity due to paralysis B. Impaired urinary elimination due to quadriplegia C. Ineffective coping due to the extent of trauma D. Ineffective airway clearance due to high cervical spinal cord injury

D. Ineffective airway clearance due to high cervical spinal cord injury Maintaining a patent airway is the most important goal for a patient with a high cervical fracture. Although all of these options are appropriate nursing diagnoses for a patient with a spinal cord injury, respiratory needs are always the highest priority. Remember the ABCs

A patient is admitted to the hospital with a C4 spinal cord injury after a motorcycle collision. The patient's blood pressure is 83/49 mm Hg and pulse is 39 beats/minute. He remains orally intubated. What is the cause of this pathophysiologic response? A. Increased vasomotor tone after the injury B. A temporary loss of sensation and flaccid paralysis below the level of injury C. Loss of parasympathetic nervous system innervation resulting in vasoconstriction D. Loss of sympathetic nervous system innervation resulting in peripheral vasodilation

D. Loss of sympathetic nervous system innervation resulting in peripheral vasodilation Neurogenic shock results from loss of vasomotor tone caused by injury, and it is characterized by hypotension and bradycardia. Loss of sympathetic nervous system innervation causes peripheral vasodilation, venous pooling, and a decreased cardiac output. These effects usually are associated with a cervical or high thoracic injury (T6 or higher).

Which sign or symptom most likely indicates that the patient's low back pain is caused by a herniated disk rather than sore back muscles? A. Pain from muscle irritation lasts for a longer period. B. The straight-leg-raising test result is negative when a herniated disk is involved. C. The patient with muscular pain often has cauda equina syndrome. D. Pain from a herniated disk typically radiates along the sciatic nerve.

D. Pain from a herniated disk typically radiates along the sciatic nerve. Radicular pain that radiates down the buttock and below the knee, along the distribution of the sciatic nerve, usually indicates disk herniation. Pain from either source can last for a long time. A positive straight-leg-raising test result indicates nerve irritation. Cauda equina syndrome is multiple nerve root compression from a herniated disk, and it is an emergency.

Which patient should be assigned to the experienced registered nurse on a neurologic floor? A. Patient with trigeminal neuralgia reporting facial pain rated at 10 B. Patient with Bell's palsy with unilateral facial droop C. Patient after surgical removal of a spinal cord tumor who is scheduled for discharge tomorrow D. Patient with traumatic injury to the cervical spinal cord who was admitted today from the emergency department

D. Patient with traumatic injury to the cervical spinal cord who was admitted today from the emergency department

Antimuscarinic poisoning can result from overdose of antihistamines, phenothiazines, and tricyclic antidepressants. Differential diagnosis is important, because antimuscarinic poisoning resembles which other condition? A. Epilepsy B. Diabetic coma C. Meningitis D. Psychosis

D. Psychosis Antimuscarinic poisoning often resembles psychosis and psychotic episodes. It is important to differentiate, because antipsychotic drugs have antimuscarinic properties and could intensify the symptoms of poisoning.

What is the best teaching for a patient who is newly diagnosed with diabetes mellitus type 2? A. Read a Snellen chart yearly. B. Be checked out for presbycusis. C. Notify the doctor if your vision has color distortion. D. See an ophthalmologist for a dilated eye examination yearly.

D. See an ophthalmologist for a dilated eye examination yearly.

You are caring for a patient admitted 1 week earlier with an acute spinal cord injury. Which assessment finding alerts you to the presence of autonomic dysreflexia? A. Tachycardia B. Hypotension C. Hot, dry skin D. Throbbing headache

D. Throbbing headache Autonomic dysreflexia is related to reflex stimulation of the sympathetic nervous system, which is reflected by hypertension, bradycardia, throbbing headache, and diaphoresis.

The nurse is caring for a patient admitted 1 week ago with an acute spinal cord injury. Which of the following assessment findings would alert the nurse to the presence of autonomic dysreflexia? A. Tachycardia B. Hypotension C. Hot, dry skin D. Throbbing headache

D. Throbbing headache Autonomic dysreflexia is related to reflex stimulation of the sympathetic nervous system reflected by hypertension, bradycardia, throbbing headache, and diaphoresis.

Which intervention should you perform in the acute care of a patient with autonomic dysreflexia? A. Administration of benzodiazepines B. Suctioning of the patient's upper airway C. Placement of the patient in the Trendelenburg position D. Urinary catheterization

D. Urinary catheterization

The nurse is caring for a patient receiving propranolol (Inderal). Which finding is most indicative of an adverse effect of this drug? A. A heart rate of 100 beats per minute B. A glucose level of 180 mg/dL C. Urinary urgency D. Wheezing

D. Wheezing Beta blockers, such as propranolol, are known to cause bronchoconstriction, which could manifest as wheezing. Other adverse effects could include bradycardia, atrioventricular (AV) heart block, heart failure, rebound cardiac excitation, inhibition of glycogenolysis, and potential central nervous system (CNS) effects.

MNEMONICS: Cholinergics/Anticholinergics ABCD'S of anticholinergics-Anorexia, Blurred vision, Constipation/Confusion, Dry mouth and Stasis of urine SLUDGE BAM (cholinergic) NCELX Salivation, Lacrimation, Urination, Diarrhea, Gastrointestinal upset/cramping and Emesis BAM-Bradycardia, Abdominal cramps, Miosis (Miny pupils)

DUMBBELLS (Cholinergic) Diarrhea Urination Bronchospasm Bradycardia Emesis Lacrimation Lethargy Salivation

One risk with the use of Sulfonylureas in the treatment of Type 2 Diabetes is ____________ ? A. Heart Failure B. Interacts wit Contrast Dye C. Hypoglycemia D. Weight Gain E. Both C and D F. All of the above

E. Both C and D A-This is TZD's aka Glitazones B-This is Metformin C-hypoglycemia is the major side effect of Sulfonylureas D-This makes sense given that it is going to cause more insulin release from pancreas and that will increase absorption of excess sugars

Immediately after spinal surgery you would want to monitor for changes in pupils? A. True B. False

False (you would want to check for changes in strength and sensation from baseline and comparing bilaterally)

Glucagon and epinephrine release are responsible for the late symptoms of hypoglycemia? A. True B. False

False-Early symptoms. Lack of glucose in the brain causes late symptoms

*A patient is found non-responsive and has a blood glucose of 50 mg/dL. IV access is NOT readily available. What do you do?*

Give IM Glucagon

Stress can cause a release a cortisol and other counter-regularatory hormones, which can cause ________ ?

HYPER-glycemia (cortisol causes decreased urinary output for one thing. There is also alpha 2 activation, which inhibits insulin release from the pancreas)

What is a retrovirus?

Its a virus that enters its host cell and uses an enzyme called reverse transcriptase to make a DNA copy of its RNA genome, which it inserts into the genome of the host cell using another enzyme called integrase. This process is REVERSE transcription because transcription is usually from DNA to RNA.

List five signs and symptoms that are present in diabetic ketoacidosis (DKA) that are not seen in hyperglycemic hyperosmolar syndrome (HHS)?

JUST REMEMBER THAT KETONES ARE NOT PRESENT AND THEY FOLLWO LOGICALLY: 1. Kussmaul Respirations (remember that the deep rapid breathing is trying to blow off CO2 due to ketoacidosis) 2. Sweet Fruity Breath (smell of ketones) 3. Decreased arterial pH and Bicarb HCO3- (the ketones are what is acidic not the hyperglycemia or hyperosmolarity) 4. Ketonemia 5. Ketonuria

Injection sites should be rotated within the same region (abdomen, thigh, or upper arm) from one injection to the next to reduce the risk of?

Lipodystrophy

You should hold this medication for 24 to 48 hours prior to a procedure using iodinated contrast dye. Resume 48 hours after procedure if lab results indicate normal kidney function?

Metformin (use insulin in mean time)

Somogyi effect is when there is hypoglycemia during the night causing stress hormones to be released that raise blood glucose levels higher. What should you do if you suspect this?

Monitor through night

KNOW THESE: Hypoglycemia Symptoms (CONTINUE OTHER SIDE) Adrenergic (early) - Hunger - Tremors or shakiness - Irritability - Anxiety - Headache - Diaphoresis - Pallor - Tachycardia - Dizziness - Tingling of the lips and extremities - Possible nausea and vomiting

Neuroglycopenic (later) - Clouding of vision - Blunted mental activity - Loss of fine motor skills - Confusion - Slurred speech - Extreme fatigue - Abnormal behavior - Pupillary sluggishness - Lossofconsciousness/Coma - Seizures

What are the goals of rehabilitation for the patient with an injury at the C6 level (select all that apply)? A. Stand erect with leg brace B. Feed self with hand devices C. Drive an electric wheelchair D. Assist with transfer activities E. Drive adapted van from wheel chair

Rehabilitation goals for a patient with a spinal cord injury at the C6 level include ability to assist with transfer and perform some self-care; feed self with hand devices; push wheelchair on smooth, flat surface; drive adapted van from wheelchair; independent computer use with adaptive equipment; and needing attendant care only for 6 hours per day.

NCLEX mnemonic for acetylcholines effects: SLUDGE Salivation Lacrimation Urinary incontinence Diarrhea Gastrointestinal cramping Emesis

SLUDGE BAM: Salivation Lacrimation Urinary incontinence Diarrhea Gastrointestinal cramping Emesis Bradycardia Abdominal cramps Miosis (miny pupils)

Differentiate between neurogenic shock and spinal shock. Are there certain levels of injury more likely to experience neurogenic or spinal shock (who is most at risk)? What are the signs of each type of shock?

Spinal shock: Temporary neurologic syndrome Characterized by • ↓ reflexes • Loss of sensation • Flaccid paralysis below level of injury Experienced by about 50% of people with acute spinal cord injury Lasts days to months SSSSSSSSSSpinal shock is SSSSSSSShort with loSS of SenSation, reflexeS and paralySiS Neurogenic shock: Loss of vasomotor tone caused by injury hypotension and bradycardia (important clinical cues) Loss of sympathetic nervous system innervation causes • Peripheral vasodilation • Venous pooling • ↓ cardiac output NNNNNeurogenic shock Never goes away and causes No iNNNNNervation of sympathetic NNNervous system

*One of the hallmarks of _______ is back pain that is exacerbated by movement and immediately relieved by sitting down?*

Stenosis

Which of the oral medications for Type 2 diabetes cause HYPOGLYCEMIA and WEIGHT GAIN?

Sulfonylureas

Decreased response of the sympathetic nervous system occurs at the level of _____ or higher?

T6

A patient with a spinal cord injury above the level of C4 would have paralysis of respiratory muscles AND all 4 extremities? T/F

TRUE (above C8/T1 so tetraplegic and above C4 so respiratory paralysis)

*Why do beta-blockers mask early symptoms of hypoglycemia but not late symptoms?*

They block adrenergic symptoms, which are caused by activation of the sympathetic nervous system (remember that the SNS mobiles glucose for fight-or-flight). They do not block the neuroglycopenic symptoms, which are caused by lack of glucose in the brain at lower levels (<50 mg/dL).

Which of the oral medications medications for type 2 diabetes causes HEART FAILURE?

Thiazolidinediones (TZDs or Glitazones) *ZONES of your heart start failing

What is the difference between spinal shock and neurogenic shock?

Think: SSSSS-pinal SHOck is SSSSShort with total loSSSS, while NeuroGenic-Never Goes away

What would be a side effect of pyridostigmine?

This is a cholinergic drug. DUMBBELLS

What is TRIUMEQ?

a commonly prescribed, fixed dose, combination medication that consists of 2 NRTI's and an INSTI?

Which of the following are anticholinergic side effects that may occur with the use of antipsychotic drugs? Select all that apply. a) Dry mouth b) Urinary retention c) Diarrhea d) Constipation e) Runny nose

a) Dry mouth, d) Constipation, b) Urinary retention Explanation: Anticholinergic side effect resulting from blockade of acetylcholine are common side effects associated with antipsychotic drugs. Others include dry mouth, slowed gastric motility, constipation, urinary hesitancy or retention, and nasal congestion. Diarrhea and a runny nose are not anticholinergic side effects.

Which statement by the patient with type 2 diabetes is accurate? a. "I am supposed to have a meal or snack if I drink alcohol." b. "I am not allowed to eat any sweets because of my diabetes." c. "I do not need to watch what I eat because my diabetes is not the bad kind." d. "The amount of fat in my diet is not important; it is just the carbohydrates that raise my blood sugar."

a. "I am supposed to have a meal or snack if I drink alcohol."

erase

a. "I should never skip doses of my medication, even if I develop side effects."

*Two days following a self-managed hypoglycemic episode at home, the patient tells the nurse that his blood glucose levels since the episode have been between 80 and 90 mg/dL. The best response by the nurse is?* a. "That is a good range for your glucose levels." b. "You should call your health care provider because you need to have your insulin increased." c. "That level is too low in view of your recent hypoglycemia, and you should increase your food intake." d. "You should only take half your insulin dosage for the next few days to get your glucose level back to normal."

a. "That is a good range for your glucose levels."

In 2010 which ethnic group is most affected by HIV the most according to CDC statistics? a. African Americans b. Whites c. Hispanic Latinos d. Asians

a. African Americans

A primary reason that the normal immune response fails to contain HIV infection is that? a. CD4+ T cells drawn to the viruses become infected and are destroyed. b. the virus inactivates B lymphocytes, preventing the production of HIV antibodies. c. natural killer cells are destroyed by the virus before the immune system can be activated. d. monocytes ingest infected cells, differentiate into macrophages, and shed viruses in body tissue

a. CD4+ T cells drawn to the viruses become infected and are destroyed.

*Urinary function during the acute phase of spinal cord injury is maintained with?* a. an indwelling catheter. b. intermittent catheterization. c. insertion of a suprapubic catheter. d. use of incontinent pads to protect the skin

a. an indwelling catheter.

During the patient's process of grieving for the losses resulting from spinal cord injury, the nurse? a. helps the patient understand that working through the grief will be a lifelong process. b. should assist the patient to move through all stages of the mourning process to acceptance. c. lets the patient know that anger directed at the staff or the family is not a positive coping mechanism. d. facilitates the grieving process so that it is completed by the time the patient is discharged from rehabilitation.

a. helps the patient understand that working through the grief will be a lifelong process.

Priority Decision: A patient is admitted to the emergency department with a possible cervical spinal cord injury following an automobile crash. During the admission of the patient, the nurse places the highest priority on? a. maintaining a patent airway. b. assessing the patient for head and other injuries. c. maintaining immobilization of the cervical spine. d. assessing the patient's motor and sensory function.

a. maintaining a patent airway.

*The nurse teaches the patient recovering from an episode of acute low back pain to?* a. perform daily exercise as a lifelong routine. b. sit in a chair with the hips higher than the knees. c. avoid occupations in which the use of the body is required. d. sleep on the abdomen or on the back with the legs extended

a. perform daily exercise as a lifelong routine.

A diabetic patient is learning to mix regular insulin and NPH insulin in the same syringe. The nurse determines that additional teaching is needed when the patient? a. withdraws the NPH dose into the syringe first. b. injects air equal to the NPH dose into the NPH vial first. c. removes any air bubbles after withdrawing the first insulin. d. adds air equal to the insulin dose into the regular vial and withdraws the dose.

a. withdraws the NPH dose into the syringe first.

Two days following a spinal cord injury, a patient asks continually about the extent of impairment that will result from the injury. The best response by the nurse is? a. "You will have more normal function when spinal shock resolves and the reflex arc returns." b. "The extent of your injury cannot be determined until the secondary injury to the cord is resolved." c. "When your condition is more stable, an MRI will be done that can reveal the extent of the cord damage." d. "Because long-term rehabilitation can affect the return of function, it will be years before we can tell what the complete effect will be."

b. "The extent of your injury cannot be determined until the secondary injury to the cord is resolved."

*Which of the following statements about metabolic side effects of ART is true (select all that apply)?* a. These are an annoying set of symptoms that are ultimately harmless. b. Body changes include central fat accumulation and peripheral wasting. c. Lipid abnormalities include increases of triglycerides and decreases in high-density cholesterol. d. Insulin resistance and hyperlipidemia can be treated with drugs to control glucose and cholesterol. e. Compared to noninfected people, insulin resistance and hyperlipidemia are difficult to treat in HIV patients.

b. Body changes include central fat accumulation and peripheral wasting. c. Lipid abnormalities include increases of triglycerides and decreases in high-density cholesterol. d. Insulin resistance and hyperlipidemia can be treated with drugs to control glucose and cholesterol.

A patient is admitted to the emergency department with a spinal cord injury at the level of T2. Which of the following findings is of most concern to the nurse? a. SpO2 of 92% b. HR of 42 beats/min c. Blood pressure of 88/60 d. Loss of motor and sensory function in arms and legs

b. HR of 42 beats/min

*To prevent hyperglycemia or hypoglycemia with exercise, the nurse teaches the patient using glucose-lowering agents that exercise should be undertaken?* a. only after a 10- to 15-g carbohydrate snack is eaten. b. about 1 hour after eating, when blood glucose levels are rising. c. when glucose monitoring reveals that the blood glucose is in the normal range. d. when blood glucose levels are high because exercise always has a hypoglycemic effect.

b. about 1 hour after eating, when blood glucose levels are rising. -Counter-regulatory hormones produced during exercise work against insulin and cause hyperglycemia

A patient with paraplegia has developed an irritable bladder with reflex emptying. The nurse teaches the patient? a. hygiene care for an indwelling urinary catheter. b. how to perform intermittent self-catheterization. c. to empty the bladder with manual pelvic pressure in coordination with reflex voiding patterns. d. that a urinary diversion, such as an ileal conduit, is the easiest way to handle urinary elimination

b. how to perform intermittent self-catheterization.

A patient with a metastatic tumor of the spinal cord is scheduled for removal of the tumor by a laminectomy. In planning postoperative care for the patient, the nurse recognizes that? a. most cord tumors cause autodestruction of the cord as in traumatic injuries. b. metastatic tumors are commonly extradural lesions that can be removed completely. c. radiation therapy is routinely administered following surgery for all malignant spinal cord tumors. d. because complete removal of intramedullary tumors is not possible, the surgery is considered palliative.

b. metastatic tumors are commonly extradural lesions that can be removed completely.

*A patient is admitted with a spinal cord injury at the C7 level. During assessment the nurse identifies the presence of spinal shock on finding?* a. paraplegia with a flaccid paralysis. b. tetraplegia with total sensory loss. c. total hemiplegia with sensory and motor loss. d. spastic tetraplegia with loss of pressure sensation.

b. tetraplegia with total sensory loss.

Goals of rehabilitation for the patient with an injury at the C6 level include (select all that apply)? a.stand erect with leg brace. b.feed self with hand devices. c.drive an electric wheelchair. d.assist with transfer activities. e.drive adapted van from wheelchair.

b.feed self with hand devices. c.drive an electric wheelchair. d.assist with transfer activities. e.drive adapted van from wheelchair.

Polydipsia and polyuria related to diabetes mellitus are primarily due to? a.the release of ketones from cells during fat metabolism. b.fluid shifts resulting from the osmotic effect of hyperglycemia. c.damage to the kidneys from exposure to high levels of glucose. d.changes in RBCs resulting from attachment of excessive glucose to hemoglobin.

b.fluid shifts resulting from the osmotic effect of hyperglycemia.

Priority Decision: The nurse is assessing a newly admitted diabetic patient. Which of these observations should be addressed as a priority by the nurse? a. Bilateral numbness of both hands b. Stage II pressure ulcer on the right heel c. Rapid respirations with deep inspiration d. Areas of lumps and dents on the abdomen

c. Rapid respirations with deep inspiration

The health care provider has ordered IV dopamine (Intropin) for a patient in the emergency department with a spinal cord injury. The nurse determines that the drug is having the desired effect when assessment findings include? a. pulse rate of 68. b. respiratory rate of 24. c. blood pressure of 106/82. d. temperature of 96.8° F (36.0° C).

c. blood pressure of 106/82. DOPAMINE DILATES?

When teaching the patient with diabetes about insulin administration, the nurse instructs the patient to? a. pull back on the plunger after inserting the needle to check for blood. b. clean the skin at the injection site with an alcohol swab before each injection. c. consistently use the same size of the appropriate strength insulin syringe to avoid dosing errors. d. rotate injection sites from arms to thighs to abdomen with each injection to prevent lipodystrophies.

c. consistently use the same size of the appropriate strength insulin syringe to avoid dosing errors.

Opportunistic diseases in HIV infection? a.are usually benign. b.are generally slow to develop and progress. c.occur in the presence of immunosuppression. d.are curable with appropriate drug interventions.

c. occur in the presence of immunosuppression.

Without surgical stabilization, immobilization and traction of the patient with a cervical spinal cord injury most frequently requires the use of? a. kinetic beds. b. hard cervical collars. c. skeletal traction with skull tongs. d. sternal-occipital-mandibular immobilizer (SOMI) brace.

c. skeletal traction with skull tongs. In lecture they were called Gardener-Wells tongs

Prophylactic measures that are routinely used as early as possible in HIV infection to prevent opportunistic and debilitating secondary problems include administration of? a. isoniazid (INH) to prevent tuberculosis. b. trimethoprim-sulfamethoxazole (TMP-SMX) for toxoplasmosis. c. vaccines for pneumococcal pneumonia, influenza, and hepatitis A and B. d. varicella-zoster immune globulin (VZIG) to prevent chickenpox or shingle

c. vaccines for pneumococcal pneumonia, influenza, and hepatitis A and B.

Which of the following patients would a nurse plan to teach how to prevent or delay the development of diabetes? a. A 62-year-old obese white man. b. An obese 50-year-old Hispanic woman. c. A child whose father has type 1 diabetes. d. A 34-year-old woman whose parents both have type 2 diabetes.

d. A 34-year-old woman whose parents both have type 2 diabetes.

When comparing the pathophysiology of type 1 and type 2 diabetes, which statement would be correct for a patient with type 2 diabetes who was admitted to the hospital with pneumonia? a. The patient must receive insulin therapy to prevent the development of ketoacidosis. b. The patient has islet cell antibodies that have destroyed the ability of the pancreas to produce insulin. c. The patient has minimal or absent endogenous insulin secretion and requires daily insulin injections. d. The patient may have sufficient endogenous insulin to prevent ketosis but is at risk for development of hyperosmolar hyperglycemic syndrome.

d. The patient may have sufficient endogenous insulin to prevent ketosis but is at risk for development of hyperosmolar hyperglycemic syndrome.

In addition to promoting the transport of glucose from the blood into the cell, insulin also? a. enhances the breakdown of adipose tissue for energy. b. stimulates hepatic glycogenolysis and gluconeogenesis. c. prevents the transport of triglycerides into adipose tissue. d. accelerates the transport of amino acids into cells and their synthesis into protein.

d. accelerates the transport of amino acids into cells and their synthesis into protein. *Insulin is an anabolic hormone. Glucagon is a catabolic hormone that causes hepatic glycogenolysis and gluconeogenesis.

A diabetic patient is found unconscious at home, and a family member calls the clinic. After determining that no glucometer is available, the nurse advises the family member to? a. try to arouse the patient to drink some orange juice. b. administer 10 U of regular insulin subcutaneously. c. call for an ambulance to transport the patient to a medical facility. d. administer glucagon 1 mg intramuscularly (IM) or subcutaneously.

d. administer glucagon 1 mg intramuscularly (IM) or subcutaneously.

In planning community education for prevention of spinal cord injuries, the nurse targets? a. elderly men. b. teenage girls. c. elementary school-age children. d. adolescent and young adult men.

d. adolescent and young adult men.

*Lispro insulin (Humalog) with NPH insulin is ordered for a patient with newly diagnosed type 1 diabetes. The nurse knows that when lispro insulin is used, it should be administered?* a. only once a day. b. 1 hour before meals. c. 30 to 45 minutes before meals. d. at mealtime or within 15 minutes of meals.

d. at mealtime or within 15 minutes of meals.

When comparing the pathophysiology of type 1 and type 2 diabetes, which statement would be correct for a patient with type 2 diabetes who was admitted to the hospital with pneumonia? a.The patient must receive insulin therapy to prevent the development of ketoacidosis. b.The patient has islet cell antibodies that have destroyed the ability of the pancreas to produce insulin. c.The patient has minimal or absent endogenous insulin secretion and requires daily insulin injections. d.The patient may have sufficient endogenous insulin to prevent ketosis but is at risk for development of hyperosmolar hyperglycemic syndrome.

d.The patient may have sufficient endogenous insulin to prevent ketosis but is at risk for development of hyperosmolar hyperglycemic syndrome.

*A patient is admitted to the ICU with a C7 spinal cord injury and diagnosed with Brown-Séquard syndrome. On physical examination, the nurse would most likely find?* a.upper extremity weakness only. b.complete motor and sensory loss below C7. c.loss of position sense and vibration in both lower extremities. d.ipsilateral motor loss and contralateral sensory loss below C7.

d.ipsilateral motor loss and contralateral sensory loss below C7.

Low back pain that radiates down the buttock along the distribution of the sciatic nerve generally indicates?

herniated interveterbral disk

ALPHA-2 agonists ______ the sympathetic nervous system?

inhibit

What is Truvada (Tenoforvir and emtricitabine)?

most commonly prescribed Pre-exposure Prophylaxis (PrEP) for HIV infection.

Myasthenia Gravis results from a decrease in acetylcholine. It is treated with the cholinergic drug?

pyridostigmine (Mestinon) M-estinon-M-yasthenia pyridostigmine is a cholin-esterase inhibitor. Any Esterase enzyme is an enzyme that breaks down something so this breaks down Ach, thus the drug increases Ach.

Why must patients infected with HIV take more than one antiviral drug?

resistance is less likely to develop

Atropine is used to treat ________ bradycardia?

symptomatic

*high risk conditions for hypoglycemia?*

• Sudden NPO status or reduction in oral intake • Enteral feeding discontinued • Total Parenteral Nutrition discontinued • Pre-meal insulin given and meal not ingested • Reduction in corticosteroid dose • Altered ability to self-report symptoms


Kaugnay na mga set ng pag-aaral

Medications for Dermatologic Conditions

View Set

Ch 15: The system of Internal Membranes (Secretion)

View Set

Day 2 Vocabulary: browser, bookmarks, & folder structure

View Set

Kidneys and Urinary System Review (powerpoints)

View Set

HDFS 2200 Complete Study Guide--Chapters 1 through 12

View Set

Psyc 523 - Chapter 3: Skills Approach

View Set

Chapter 26: Nursing Management: Upper Respiratory Problems

View Set